Vous êtes sur la page 1sur 3406

Q0001:What gland is found in the muscular triangle of the

neck?
Thyroid gland
Q0002:Is an afferent or efferent pupillary defect described as
B/L pupillary constriction when light is shined in the
unaffected eye and B/L paradoxical dilation when light is
shined in the affected eye?
Afferent pupillary defect (CN II lesion); in an efferent
pupillary defect (CN III); B/L constrict when light is shined in
the unaffected eye and consentual pupil constriction occurs
when light is shined in the affected eye.
Q0003:What is the name of the spinal cord passing within the
subarachnoid space and forming the spinal nerves that exit the
lumbar and sacral foramina?
Cauda equina
Q0004:Name the laryngeal muscle described by the
following;? Pulls the arytenoids cartilages closer to the
thyroid; relaxing the vocal ligaments and thereby decreasing
the pitch
Thyroarytenoid muscles
Q0005:Name the laryngeal muscle described by the
following;? Tenses the vocal ligaments; increasing the distance
between the cartilages; thereby increasing the pitch
Cricothyroid muscles
Q0006:Name the laryngeal muscle described by the
following;? Adducts the vocal ligaments; closes the air
passageway during swallowing; and allows phonation
Lateral cricoarytenoid muscles
Q0007:Name the laryngeal muscle described by the
following;? Only muscle to abduct the vocal cords
Posterior cricoarytenoid muscles
Q0008:Where does the parotid (Stensen's) duct enter the oral
cavity?
Opposite the second upper molar tooth
Q0009:From what aortic arch are the following structures
derived?;? Common and internal carotid arteries
Third aortic arch;MS CARD is my mnemonic for the aortic
arch derivatives
Q0010:From what aortic arch are the following structures
derived?;? Degenerates
Fifth;MS CARD is my mnemonic for the aortic arch
derivatives
Q0011:From what aortic arch are the following structures
derived?;? Stapes artery
Second;MS CARD is my mnemonic for the aortic arch
derivatives
Q0012:From what aortic arch are the following structures
derived?;? Maxillary artery
First;MS CARD is my mnemonic for the aortic arch
derivatives
Q0013:From what aortic arch are the following structures
derived?;? Arch of the aorta and right subclavian artery
Fourth;MS CARD is my mnemonic for the aortic arch
derivatives
Q0014:From what aortic arch are the following structures
derived?;? Right and left pulmonary arteries and the ductus
arteriosus
Sixth;MS CARD is my mnemonic for the aortic arch
derivatives
Q0015:What abdominal muscle contributes to the anterior
layer of the rectus sheath; forms the inguinal ligament; and in
men gives rise to the external spermatic fascia of the spermatic
cord?
External abdominal oblique
Q0016:Name the compartment of the lower extremity and the
nerve based on its movements;? Adduct the thigh and flex the
hip
Medial compartment of the thigh; obturator nerve
Q0017:Name the compartment of the lower extremity and the
nerve based on its movements;? Plantar flex the foot; flex the
toes; and invert the foot
Posterior compartment of the leg; tibial nerve
Q0018:Name the compartment of the lower extremity and the
nerve based on its movements;? Dorsiflex the foot; extend the
toes; and invert the foot
Anterior compartment of the leg; deep peroneal nerve
Q0019:Name the compartment of the lower extremity and the
nerve based on its movements;? Flex the hip and extend the
knee
Anterior compartment of the thigh; femoral nerve
Q0020:Name the compartment of the lower extremity and the
nerve based on its movements;? Extend the hip and flex the
knee
Posterior compartment of the thigh; tibial nerve
Q0021:Name the compartment of the lower extremity and the
nerve based on its movements;? Plantar flex the foot and evert
the foot
Lateral compartment of the leg; superficial peroneal nerve
Q0022:What are the five branches of the posterior cord of the
brachial plexus?
STARS;1. Upper Subscapularis ;2. Thoracodorsal ;3. Axillary
;4. Radial ;5. Lower Subscapularis
Q0023:Name the correct artery;? The right recurrent laryngeal
nerve passes around it.
Right brachiocephalic artery
Q0024:Name the correct artery;? The left recurrent laryngeal
nerve passes around it.
Arch of the aorta
Q0025:The inferior mesenteric vein drains into it.
The splenic vein
Q0026:Are the quadrate and caudate lobes of the liver
functionally part of the left or right lobe?
Functionally they are part of the left lobe of the liver because
they receive their blood supply from the left hepatic artery.
Anatomically they are considered part of the right lobe of the
liver.
Q0027:What bones make up the acetabulum?
Pubis; ilium; and ischium
Q0028:What is the anatomic positioning of the right and left
gastric nerve plexus of the esophagus as they pass through the
diaphragm?
LARP: Left goes Anterior and Right goes Posterior (because
of the rotation of the gut; remember your embryology!)
Q0029:What vessel is lacerated in an epidural hematoma?
Middle meningeal artery
Q0030:True or false? Below the arcuate line; all the
aponeurotic fibers run anterior to the rectus abdominis.
True
Q0031:What ocular muscle;? Adducts the eyeball and is
involved in horizontal conjugate gaze?
Medial rectus (CN III);(LR6 SO4)3
Q0032:What ocular muscle;? Elevates and adducts the
eyeball?
Superior rectus (CN III);(LR6 SO4)3
Q0033:What ocular muscle;? Depresses and abducts the
eyeball?
Superior Oblique (CN IV);(LR6 SO4)3
Q0034:What ocular muscle;? Elevates and abducts the
eyeball?
Inferior Oblique (CN III);(LR6 SO4)3
Q0035:What ocular muscle;? Abducts the eyeball and is
involved in horizontal conjugate gaze?
Lateral rectus (CN VI);(LR6 SO4)3
Q0036:What ocular muscle;? Depresses and adducts the
eyeball?
Inferior rectus (CN III) ;(LR6 SO4)3
Q0037:Which muscles of the eye are under parasympathetic
control?
Constrictor pupillae and ciliary muscles
Q0038:Which direction does the uvula deviate in a left vagus
nerve lesion?
A left CN X lesion results in the uvula deviating to the right.
(Uvula points away from the affected side.)
Q0039:Is a subdural hematoma an arterial or venous bleed?
Subdural hematoma is a rupture of the cerebral veins where
they enter the superior sagittal sinus.
Q0040:Which CNs are found in the midline of the brainstem?
CN I; II; III; VI; and XII;Add 1 + 1 = 2; 1 + 2 = 3; 1 + 2 + 3 =
6; 1 + 2 + 3 + 6 = 12
Q0041:What muscles insert in or on the intertubercular groove
of the humerus?
"Lady between two Majors": latissimus dorsi; pectoralis
major; and teres major
Q0042:What nerve supplies taste sensation to the anterior
two-thirds of the tongue?
Chorda tympani of CN VII
Q0043:What part of the heart forms;? The right border?
Right atrium
Q0044:What part of the heart forms;? Left border?
Left ventricle and auricle of left atrium
Q0045:What part of the heart forms;? Apex?
Tip of the left ventricle
Q0046:What part of the heart forms;? Base?
Left atrium and tip of the right atrium
Q0047:What part of the heart forms;? Superior border?
Conus arteriosus of the right ventricle and right and left
auricles
Q0048:What part of the heart forms;? Anterior wall?
Right ventricle
Q0049:What part of the heart forms;? Posterior wall?
Left atrium
Q0050:What part of the heart forms;? Diaphragmatic wall?
Left ventricle and tip of right ventricle
Q0051:What nerves carry the sensory and motor components
of the blink reflex?
CN V1 carries the sensory and CN VII carries the motor
component of the blink reflex.
Q0052:What muscle keeps the stapes taut against the oval
window?
Stapedius muscle
Q0053:Name the components of the femoral canal; working
laterally to medially.
NAVEL: Femoral Nerve; Artery; Vein; Empty space; and
Lymphatics/Lacunar ligament
Q0054:What muscle is most superior in the orbit?
Levator palpebrae superioris
Q0055:What portion of the pericardium adheres to the tunica
adventitia of the great vessels?
Fibrous pericardium
Q0056:What two veins form the portal vein?
The superior mesenteric vein and the splenic vein (after it
receives the inferior mesenteric vein) join to form the portal
vein.
Q0057:What CNs are responsible for the sensor and motor
components of the light reflex?
CN II is the sensory limb and CN III is the motor component
through parasympathetic stimulation.
Q0058:Arrange the following layers in the correct sequence
through which a needle must pass in a lumbar puncture;? Skin
;? Subarachnoid space ;? Interspinous ligament ;? Dura mater
;? Deep fascia ;? Epidural space ;? Superficial fascia ;?
Interlaminar space ;? Supraspinous ligament ;? Arachnoid
mater
During a lumbar puncture the needle passes through the
interlaminar space in the midline of L3–L4; with the tip of the
iliac crest in the flexed position as the landmark;Order of
puncture;1. Skin ;2. Superficial fascia ;3. Deep fascia ;4.
Supraspinous ligament ;5. Interspinous ligament ;6.
Interlaminar space ;7. Epidural space ;8. Dura mater ;9.
Arachnoid mater ;10. Subarachnoid space;;(They ask this in
some variation every year; so know it.)
Q0059:What ocular ganglion is affected if the pupil on the
affected side sluggishly responds to light with normal
accommodation?
Ciliary ganglion producing a tonic pupil
Q0060:What is the name for the most prominent spinous
process?
Vertebra prominens (C7 in 70% of cases; C6 in 20%; T1 in
10%)
Q0061:What muscles make up the rotator cuff?
SITS—Subscapularis; Infraspinatus; Teres minor;
Supraspinatus
Q0062:What is the function of white rami communicantes?
They are preganglionic sympathetic axons. They are white
because they are myelinated.
Q0063:What muscle or muscles are innervated by the
following nerves?;? Suprascapular nerve
Supraspinatus and infraspinatus
Q0064:What muscle or muscles are innervated by the
following nerves?;? Upper subscapularis nerve
Subscapularis
Q0065:What muscle or muscles are innervated by the
following nerves?;? Thoracodorsal nerve
Latissimus dorsi
Q0066:What muscle or muscles are innervated by the
following nerves?;? Long thoracic nerve
Serratus anterior
Q0067:What nerve is associated with the following
functions?;? Flex the wrist and digits; pronate the wrist and
the LOAF (Lumbricales; Opponens pollicis; Abductor
pollicis brevis; Flexor pollicis brevis) muscles of the hand
Median nerve
Q0068:What nerve is associated with the following
functions?;? Flex the shoulder; flex the elbow; and supinate
the elbow
Musculocutaneous nerve
Q0069:What nerve is associated with the following
functions?;? Innervation of the flexor carpi ulnaris; flexor digiti
profundus (pinky and ring fingers); and the intrinsic muscles
of the hand
Ulnar nerve
Q0070:What nerve is associated with the following
functions?;? Supinate the wrist; extend the wrist and digits;
extend the shoulder and elbow
Radial nerve
Q0071:What abdominal muscle runs horizontally; contributes
to the posterior rectus sheath; and contributes to form the
conjoint tendon?
Transverse abdominis
Q0072:Which CNs act as the sensory and motor components
of the gag reflex?
The sensory limb is via CN IX; and the motor limb is from
CN X.
Q0073:Which kidney is lower? Why?
The right kidney is lower in the abdominal cavity because of
the amount of space the liver occupies.
Q0074:What two regions of the vertebral column are
considered primary curvatures?
Thoracic and sacral
Q0075:What vein drains the lower third of the thoracic wall?
Hemiazygous vein
Q0076:At what point does the axillary artery become the
brachial artery?
When it crosses the teres major
Q0077:What direction would the tongue protrude in a left CN
XII lesion?
Left CN XII lesion would result in the tongue pointing to the
left (points at the affected side).
Q0078:At what vertebral level does the common carotid
artery bifurcate?
C4 (the upper border of the thyroid cartilage)
Q0079:True or false? Males are more likely to develop
femoral hernias than females.
False. Females are more likely to develop femoral hernias then
males (remember Female's Femoral).
Q0080:In what compartment of the thigh is the profundus
femoris artery found?
Anterior compartment (it's the blood supply to the posterior
compartment)
Q0081:Where is the cupola of the lung in relation to the
subclavian artery and vein?
The cupola of the lung is posterior to the subclavian artery
and vein. It is the reason one must be cautious when
performing subclavian venipuncture.
Q0082:True or false? The first cervical vertebra has no
vertebral body.
True. The odontoid process of C2 acts as the vertebral body
of C1 allowing lateral rotation of the head.
Q0083:What is the largest muscle in the body?
Gluteus maximus
Q0084:At what vertebral levels does the aortic arch begin and
end?
It both begins and ends at T4 (sternal angle [of Louis]).
Q0085:What artery travels with the following veins?;? Great
cardiac vein
Left anterior descending artery
Q0086:What artery travels with the following veins?;? Middle
cardiac vein
Posterior interventricular artery
Q0087:What artery travels with the following veins?;? Small
cardiac vein
Right coronary artery
Q0088:The ophthalmic artery is a branch of what vessel?
Internal carotid artery
Q0089:What structure or structures cross the diaphragm at;?
T8 level?
IVC;Remember: 1 at T8; 2 at T10; and 3 at T12
Q0090:What structure or structures cross the diaphragm at;?
T10 level?
Esophagus and esophageal nerve plexus (CN X);Remember: 1
at T8; 2 at T10; and 3 at T12
Q0091:What structure or structures cross the diaphragm at;?
T12 level?
Aorta; azygos vein; and thoracic duct;Remember: 1 at T8; 2 at
T10; and 3 at T12
Q0092:Is the carotid sinus sensitive to pressure or oxygen?
The carotid sinus is a pressure-sensitive (low) receptor; while
the carotid body is an oxygen-sensitive (low) receptor.
(Remember "Sinus Pressure").
Q0093:What nerve or nerves supply general sensation and
taste to the posterior third of the tongue?
CN IX and X
Q0094:Which muscle of the eye is under sympathetic
control?
Dilator pupillae muscle
Q0095:True or false? both the left and right lungs have an
oblique fissure?
True. on the right lung the oblique fissure divides the middle
from the inferior lobe and the horizontal fissure further
divides the middle from the upper lobe. On the left the
oblique divides the superior from the inferior lobe.
Q0096:What are the three branches of the lateral cord of the
brachial plexus?
1. Lateral pectoral ;2. Lateral head of the median ;3.
Musculocutaneus
Q0097:What is the major difference between the veins in the
face and the veins in the rest of the body?
There are no valves and no smooth muscle in the walls of the
veins in the face.
Q0098:Name the bony articulations of the following sites. Be
specific;? Shoulder
Clavicle; acromion; and glenoid fossa of the scapula and the
humerus
Q0099:Name the bony articulations of the following sites. Be
specific;? Elbow
Humerus with ulna (major) and radius (minor)
Q0100:Name the bony articulations of the following sites. Be
specific;? Wrist
Radius with scaphoid and lunate and ulna with triquetrum and
pisiform (Remember; for major articulations; wrist/radius and
humerus/ulna = elbow)
Q0101:What is the only laryngeal muscle innervated by the
external laryngeal nerve?
Cricothyroid muscle; all other laryngeal muscles are
innervated by the recurrent laryngeal nerve.
Q0102:What seven structures are found in more than one
mediastinum?
Esophagus; SVC; vagus nerve; azygos vein; thoracic duct;
thymus; and phrenic nerve
Q0103:How many bronchopulmonary segments are on the
right lung? Left lung?
There are 10 bronchopulmonary segments on the right and 8
on the left.
Q0104:The duodenal–jejunal flexure is suspended from the
posterior abdominal wall by what?
Ligament of Treitz
Q0105:What is the only tongue muscle innervated by CN X?
Palatoglossus muscle is innervated by CN X; all other tongue
muscles are innervated by CN XII.
Q0106:What abdominal muscle runs in a posteroinferior
direction; splits to contribute to the rectus sheath; contributes
to the formation of the conjoint tendon; and in men gives rise
to the middle spermatic fascia and the cremasteric muscle of
the spermatic cord?
Internal abdominal oblique
Q0107:What are the five branches of the superior mesenteric
artery?
Inferior pancreaticoduodenal; middle colic; right colic;
ileocolic; and 10 to 15 intestinal arteries
Q0108:What spinal nerves contribute to the pelvic splanchnic
(parasympathetic) nerves that innervate the detrusor muscle
of the urinary bladder?
S2; S3; S4—keeps the pee-pee off the floor!
Q0109:What connects the third and the fourth ventricles?
Cerebral aqueduct
Q0110:What nerve and artery could be affected in a humeral
neck fracture?
Axillary nerve and posterior humeral artery
Q0111:What type of hernia is described as passing through
the deep lateral ring of the inguinal canal?
Indirect hernia passes in the inguinal canal; a direct hernia
passes directly through Hesselbach's triangle.
Q0112:What two vessels come together to form the external
jugular vein?
1. Posterior auricular vein ;2. Posterior division of the
retromandibular vein
Q0113:What is the only vein in the body with a high O2
content?
The pulmonary vein; which carries oxygenated blood from the
lung to the left atrium.
Q0114:What are the three branches of the celiac trunk?
The left gastric; splenic; and common hepatic arteries
Q0115:What region of the pharynx does the eustachian tube
enter?
Nasopharynx
Q0116:What is the only muscle of the soft palate that is
innervated by CN V3?
The tensor veli palatine is innervated by the mandibular
division of the trigeminal nerve; all others are innervated by
CN X.
Q0117:How many pairs of spinal nerves exit from the spinal
cord?
31 pairs
Q0118:What artery turns into the dorsalis pedis when it
crosses the extensor retinaculum?
Anterior tibial artery
Q0119:What is the term for pupils that react normally to
accommodation but have bilateral loss of constriction in
response to light?
Argyll Robertson pupils
Q0120:What connects the lateral ventricles to the third
ventricle?
Foramen of Monro
Q0121:What nerve supplies general sensation to the anterior
two-thirds of the tongue?
Lingual nerve of CN V3
Q0122:What type of pleura is adherent to the surface of the
organ?
Visceral pleura
Q0123:What artery supplies the left ventricle; left atrium; and
interventricular septum?
Left coronary artery
Q0124:Where are the tonsillar tissues?
Waldeyer's ring
Q0125:What is the name of the superficial subcutaneous
fascia of the abdomen containing fat?
Camper's fascia; Scarpa's fascia is devoid of fat. (Remember
campers are fat.)
Q0126:What are the three anatomic characteristics that
differentiate the large bowel from the small bowel and the
rectum?
1. Tinea coli ;2. Haustra ;3. Epiploic appendages
Q0127:What area of the posterior aspect of the eye has no
photoreceptors?
The optic disk is the blind spot.
Q0128:At the level of rib 6; the internal thoracic artery
divides into what two arteries?
Musculophrenic and superior epigastric arteries
Q0129:What is the name of inflammation of the prepatellar
bursa?
Housemaid's knee
Q0130:What nerve roots constitute the cervical plexus?
C1 through C4
Q0131:Name the compartment of the mediastinum associated
with the following thoracic structures;? Heart and pericardium
Middle
Q0132:Name the compartment of the mediastinum associated
with the following thoracic structures;? Descending aorta
Posterior
Q0133:Name the compartment of the mediastinum associated
with the following thoracic structures;? Thymus
Superior and anterior
Q0134:Name the compartment of the mediastinum associated
with the following thoracic structures;? Phrenic nerve
Superior and middle
Q0135:Name the compartment of the mediastinum associated
with the following thoracic structures;? Esophagus
Superior and posterior
Q0136:Name the compartment of the mediastinum associated
with the following thoracic structures;? Trachea
Superior
Q0137:Name the compartment of the mediastinum associated
with the following thoracic structures;? Ascending aorta
Middle
Q0138:Name the compartment of the mediastinum associated
with the following thoracic structures;? Thoracic duct
Superior and posterior
Q0139:Name the compartment of the mediastinum associated
with the following thoracic structures;? Azygos vein
Superior and posterior
Q0140:Name the compartment of the mediastinum associated
with the following thoracic structures;? SVC
Superior and middle
Q0141:Name the compartment of the mediastinum associated
with the following thoracic structures;? Splanchnic nerves
Posterior
Q0142:Name the compartment of the mediastinum associated
with the following thoracic structures;? Aortic arch
Superior
Q0143:Name the compartment of the mediastinum associated
with the following thoracic structures;? IVC
Middle
Q0144:Name the compartment of the mediastinum associated
with the following thoracic structures;? Vagus nerve
Posterior
Q0145:Name the compartment of the mediastinum associated
with the following thoracic structures;? Brachiocephalic vein
Superior
Q0146:Name the compartment of the mediastinum associated
with the following thoracic structures;? Pulmonary artery and
veins
Middle
Q0147:Name the compartment of the mediastinum associated
with the following thoracic structures;? Left common carotid
artery
Superior
Q0148:Name the compartment of the mediastinum associated
with the following thoracic structures;? Left subclavian artery
Superior
Q0149:What is the only organ in the body supplied by
preganglionic sympathetic fibers?
Adrenal medulla
Q0150:The left subclavian artery is a branch of what artery?
The left is a branch of the aortic arch; while the right is a
branch of the brachiocephalic trunk.
Q0151:What are the four muscles of mastication?
1. Masseter ;2. Temporalis ;3. Medial pterygoid ;4. Lateral
pterygoid
Q0152:With what thoracic vertebra or vertebrae does rib 7
articulate?
Rib 7 articulates with T7 and T8. Each rib articulates with the
corresponding numerical vertebral body and the vertebral
body below it.
Q0153:What are the three branches of the inferior mesenteric
artery?
Left colic; superior rectal; and sigmoidal arteries
Q0154:What is the only valve in the heart with two cusps?
Mitral (bicuspid) valve
Q0155:What are five clinical signs of portal HTN?
Caput medusa; internal hemorrhoids; esophageal varices;
retroperitoneal varices; and splenomegaly
Q0156:What three muscles constitute the erector spinae?
1. Iliocostalis ;2. Longissimus ;3. Spinalis ;;("I Love Science"
muscles)
Q0157:What nerve is compromised in carpal tunnel
syndrome?
Median nerve
Q0158:What vascular injury may result from a supracondylar
fracture of the femur?
The popliteal artery; the deepest structure in the popliteal
fossa; risks injury in a supracondylar fracture of the femur.
Q0159:What nerve and artery could be affected in a midshaft
humeral fracture?
Radial nerve and the profunda brachii artery
Q0160:Name the 10 retroperitoneal organs.
1. Duodenum (all but the first part) ;2. Pancreas ;3. Ascending
Colon ;4. Descending colon;5. Rectum;6. Supra renal glands
(adrenals) ;7. Kidneys ;8. Ureters ;9. Aorta ;10. IVC ;;D
CUPS DAKRI is the mnemonic; everything else is covered
with peritoneum
Q0161:Ventral rami of what cervical nerves constitute the
phrenic nerve?
C3; C4; and C5 keep the diaphragm alive!
Q0162:What is the region of the fallopian tube where
fertilization most commonly occurs?
Ampulla
Q0163:What foramen must be traversed for entry into the
lesser peritoneal sac?
Foramen of Winslow
Q0164:Name the structure that enters or exits the following
foramina;? Foramen magnum
CN XI; vertebral arteries
Q0165:Name the structure that enters or exits the following
foramina;? Foramen spinosum
Middle meningeal artery
Q0166:Name the structure that enters or exits the following
foramina;? Foramen rotundum
CN V2
Q0167:Name the structure that enters or exits the following
foramina;? Foramen ovale
CN V3 and the lesser petrosal nerve
Q0168:Name the structure that enters or exits the following
foramina;? Jugular foramen
CN IX; X; and XI; sigmoid sinus
Q0169:Name the structure that enters or exits the following
foramina;? Carotid canal
Internal carotid artery and sympathetic plexus
Q0170:Name the structure that enters or exits the following
foramina;? Stylomastoid foramen
CN VII
Q0171:Name the structure that enters or exits the following
foramina;? Hypoglossal canal
CN XII
Q0172:Name the structure that enters or exits the following
foramina;? Internal auditory meatus
CN VII and VIII
Q0173:Name the structure that enters or exits the following
foramina;? Optic canal
CN II and ophthalmic artery
Q0174:Name the structure that enters or exits the following
foramina;? Cribriform plate
CN I
Q0175:What vessel can be found atop the scalene anterior?
Subclavian vein
Q0176:What component of the corneal reflex is lost in a CN
VII deficit?
Motor aspect
Q0177:A motor lesion to the right CN V results in deviation
of the jaw to which side?
A right CN V lesion results in weakened muscles of
mastication; and the jaw deviates to the right.
Q0178:What two arteries join to form the superficial and deep
palmar arches of the hand?
Ulnar and radial arteries (ulnar is the main supplier)
Q0179:What two ligaments of the uterus are remnants of the
gubernaculum?
Round and ovarian ligaments
Q0180:What segments of the lumbosacral plexus form the
following nerves?;? Tibial nerve
L4 to S3;(L2 to L4; thigh; L4 to S3; leg)
Q0181:What segments of the lumbosacral plexus form the
following nerves?;? Common peroneal nerve
L4 to S3;(L2 to L4; thigh; L4 to S3; leg)
Q0182:What segments of the lumbosacral plexus form the
following nerves?;? Femoral nerve
L2 to L4;(L2 to L4; thigh; L4 to S3; leg)
Q0183:What segments of the lumbosacral plexus form the
following nerves?;? Obturator nerve
L2 to L4;(L2 to L4; thigh; L4 to S3; leg)
Q0184:What three structures are in contact with the left colic
flexure? With the right colic flexure?
Left: stomach; spleen; and left kidney;;right: liver; duodenum;
and right kidney
Q0185:What three muscles constitute the pes anserinus?
1. Sartorius ;2. Gracilis ;3. Semitendinous
Q0186:What is the only pharyngeal muscle not innervated by
CN X?
Stylopharyngeus muscle is innervated by CN IX; all other
pharyngeal muscles are innervated by CN X.
Q0187:What vessels carry deoxygenated blood into the lungs
from the right ventricle?
The right and left pulmonary arteries; the only arteries that
carry deoxygenated blood
Q0188:Fracture of the fibular neck; resulting in foot drop; is
an injury of what nerve?
Common peroneal nerve
Q0189:What vein is formed by the union of the right and left
brachiocephalic veins?
Superior vena cava
Q0190:If inserting a needle to perform a pleural tap or
insertion of a chest tube; do you use the inferior or the
superior border of a rib as your landmark? Why?
The superior border of the inferior intercostal rib is your
landmark for a pleural tap because along the inferior border of
each rib is the neurovascular bundle; and you would risk
injury if you went below the rib.
Q0191:What muscle laterally rotates the femur to unlock the
knee?
Popliteus
Q0192:What chamber of the eye lies between the iris and the
lens?
Posterior chamber
Q0193:What artery supplies the right atrium; right ventricle;
sinoatrial and atrioventricular nodes?
Right coronary artery
Q0194:What four branches of the brachial plexus arise prior
to the first rib?
1. Dorsal scapular ;2. Suprascapular ;3. Long thoracic ;4.
Nerve to subclavius
Q0195:What vertebral level is marked by the xiphoid process?
T9
Q0196:What lower extremity nerve is described by the
following motor loss?;? Loss of eversion; inversion;
dorsiflexion; and plantarflexion of the foot
Common peroneal nerve
Q0197:What lower extremity nerve is described by the
following motor loss?;? Loss of flexion of the knees and toes;
plantarflexion; and weakened inversion
Tibial nerve
Q0198:What lower extremity nerve is described by the
following motor loss?;? Loss of knee extension; weakened hip
flexion
Femoral nerve
Q0199:What lower extremity nerve is described by the
following motor loss?;? Loss of abduction of the hip resulting
in Trendelenburg gait
Superior gluteal nerve
Q0200:What lower extremity nerve is described by the
following motor loss?;? Loss of flexion of the knee and all
function below the knee; weakened extension of the thigh
Sciatic nerve
Q0201:What lower extremity nerve is described by the
following motor loss?;? Loss of adduction of the thigh
Obturator nerve
Q0202:What nerve lesion presents with ape or simian hand as
its sign?
Median nerve lesion
Q0203:What muscle acts in all ranges of motion of the arm?
Deltoid
Q0204:What is the first branch of the abdominal aortic artery?
Inferior phrenic artery
Q0205:What vessel does the right gonadal vein drain into?
The right gonadal vein drains into the inferior vena cava
directly; and the left gonadal vein drains into the left renal
vein.
Q0206:What two muscles do you test to see whether CN XI
is intact?
Trapezius and sternocleidomastoid
Q0207:What two CNs are responsible for the carotid body
and sinus reflexes?
CN IX and X
Q0208:At what vertebral level does the trachea bifurcate?
T4 vertebral level posteriorly and anteriorly at the sternal
angle (angle of Louis).
Q0209:What is the function of the arachnoid granulations?
Resorb CSF into the blood
Q0210:Damage to what nerve will give you winged scapula?
Long thoracic nerve. To avoid confusing long thoracic nerve
and lateral thoracic artery: long has an n for nerve; lateral has
an a for artery.
Q0211:What portion of the intervertebral disk is a remnant of
the notochord?
Nucleus pulposus
Q0212:What component of the pelvic diaphragm forms the
rectal sling (muscle of continence)?
Puborectalis
Q0213:What are the five branches of the median cord of the
brachial plexus?
Four Ms and a U ;1. Median ;2. Medial antebrachial ;3.
Medial pectoral ;4. Medial brachial cutaneus ;5. Ulnar
Q0214:What bone houses the ulnar groove?
Humerus (between the medial epicondyle and the trochlea)
Q0215:What CN is associated with the sensory innervation
of;? Nasopharynx?
Maxillary division of CN V and glossopharyngeal nerves
Q0216:What CN is associated with the sensory innervation
of;? Oropharynx?
Glossopharyngeal nerve
Q0217:What CN is associated with the sensory innervation
of;? Laryngopharynx?
Vagus nerve
Q0218:What protective covering adheres to the spinal cord
and CNS tissue?
Pia mater
Q0219:What is the name of the urinary bladder where the
ureters enter and the urethra exits?
Urinary trigone
Q0220:What is the term when the brachial artery is
compressed; resulting in ischemic contracture of the hand?
Volkmann's contracture
Q0221:What attaches the cusps of the valves to the papillary
muscles in the heart?
Chordae tendineae
Q0222:What is the lymphatic drainage of the pelvic organs?
Internal iliac nodes
Q0223:What bursa is inflamed in clergyman's knee?
Infrapatellar bursa
Q0224:What muscle is the chief flexor of the hip?
Psoas major
Q0225:What component of the ANS; when stimulated;
results in bronchoconstriction?
Parasympathetic stimulation; via the vagus nerve; results in
bronchoconstriction; whereas sympathetic stimulation results
in bronchodilation.
Q0226:What muscles in the hand adduct the fingers?
The Palmar interosseus ADducts; whereas the Dorsal
interosseus ABducts (PAD and DAB)
Q0227:What type of cerebral bleed is due to a rupture of a
berry aneurysm in the circle of Willis?
Subarachnoid hematoma
Q0228:What are the five terminal branches of the facial nerve?
1. Temporal ;2. Zygomatic ;3. Buccal ;4. Mandibular ;5.
Cervical ;;(Two Zebras Bit My Clavicle.)
Q0229:What structure of the knee is described thus?;? C-
shaped shock absorber; aids in attachment of the tibia to the
femur via the medial collateral ligament
Medial meniscus
Q0230:What structure of the knee is described thus?;?
Prevents posterior displacement and has medial-to-lateral
attachment on the tibia
Posterior cruciate ligament
Q0231:What structure of the knee is described thus?;?
Prevents adduction
Lateral collateral ligament
Q0232:What structure of the knee is described thus?;?
Prevents anterior displacement and has lateral-to-medial
attachment on the tibia
ACL
Q0233:What structure of the knee is described thus?;?
Prevents abduction
Medial collateral ligament
Q0234:What branches of CN X are the sensory and motor
components of the cough reflex? Be specific.
The sensory component is through the superior laryngeal
nerve; and the motor limb is via the recurrent laryngeal nerve.
Q0235:What nerves provide sensory innervation above the
vocal cords? Below the vocal cords?
The internal laryngeal nerve supplies sensory information
from above the vocal cords while the recurrent laryngeal nerve
supplies sensory information below.
Q0236:From what pharyngeal groove is the external auditory
meatus derived?
First pharyngeal groove; all others degenerate.
Q0237:What embryonic structure forms the following adult
male structures?;? Corpus cavernosus; corpus spongiosum;
and glans and body of the penis
Phallus
Q0238:What embryonic structure forms the adult male
structure?;? Scrotum
Labioscrotal swelling
Q0239:What embryonic structure forms the adult male
structure?;? Urinary bladder; urethra; prostate gland;
bulbourethral gland
Urogenital sinus
Q0240:What embryonic structure forms the adult male
structure?;? Testes; seminiferous tubules; and rete testes
Gonads
Q0241:What embryonic structure forms the adult male
structure?;? Ventral part of the penis
Urogenital folds
Q0242:What embryonic structure forms the adult male
structure?;? Gubernaculum testes
Gubernaculum
Q0243:What embryonic structure forms the adult male
structure?;? Epididymis; ductus deferens; seminal vesicle; and
ejaculatory duct
Mesonephric duct
Q0244:Which PG is associated with maintaining a PDA?
PGE and intrauterine or neonatal asphyxia maintain patency
of the ductus arteriosus. Indomethacin; ACh; and
catecholamines promote closure of the ductus arteriosus.
Q0245:When does the primitive gut herniate out of the
embryo?;When does it go back into the embryo?
6 weeks;10 weeks
Q0246:What results when the palatine prominences fail to
fuse with the other side?
Cleft palate
Q0247:What is the term for a direct connection between the
intestine and the external environment through the umbilicus
because the vitelline duct persists?
Vitelline fistula
Q0248:Where do the primordial germ cells arise?
From the wall of the yolk sac
Q0249:What disorder is due to a 5--reductase deficiency;
resulting in testicular tissue and stunted male external
genitalia?
Male pseudo-intersexuality (hermaphrodite); these individuals
are 46XY.
Q0250:Does the zygote divide mitotically or meiotically?
The zygote divides mitotically; only germ cells divide
meiotically.
Q0251:During what embryonic week does the intraembryonic
coelom form?
Third week
Q0252:Name the primary vesicle the following structures are
derived from (proencephalon; mesencephalon; or
rhombencephalon);? Cerebral hemispheres
Proencephalon
Q0253:Name the primary vesicle the following structures are
derived from (proencephalon; mesencephalon; or
rhombencephalon);? Midbrain
Mesencephalon
Q0254:Name the primary vesicle the following structures are
derived from (proencephalon; mesencephalon; or
rhombencephalon);? Cerebellum
Rhombencephalon
Q0255:Name the primary vesicle the following structures are
derived from (proencephalon; mesencephalon; or
rhombencephalon);? Medulla
Rhombencephalon
Q0256:Name the primary vesicle the following structures are
derived from (proencephalon; mesencephalon; or
rhombencephalon);? Diencephalon
Proencephalon
Q0257:Name the primary vesicle the following structures are
derived from (proencephalon; mesencephalon; or
rhombencephalon);? Metencephalon
Rhombencephalon
Q0258:Name the primary vesicle the following structures are
derived from (proencephalon; mesencephalon; or
rhombencephalon);? Telencephalon
Proencephalon
Q0259:Name the primary vesicle the following structures are
derived from (proencephalon; mesencephalon; or
rhombencephalon);? Thalamus
Proencephalon
Q0260:Name the primary vesicle the following structures are
derived from (proencephalon; mesencephalon; or
rhombencephalon);? Eye
Proencephalon*;*diencephalon derivative
Q0261:Name the primary vesicle the following structures are
derived from (proencephalon; mesencephalon; or
rhombencephalon);? Pons
Rhombencephalon
Q0262:Name the primary vesicle the following structures are
derived from (proencephalon; mesencephalon; or
rhombencephalon);? Myelencephalon
Rhombencephalon
Q0263:Name the primary vesicle the following structures are
derived from (proencephalon; mesencephalon; or
rhombencephalon);? Pineal gland
Proencephalon*;*diencephalon derivative
Q0264:Name the primary vesicle the following structures are
derived from (proencephalon; mesencephalon; or
rhombencephalon);? Cerebral aqueduct
Mesencephalon
Q0265:Name the primary vesicle the following structures are
derived from (proencephalon; mesencephalon; or
rhombencephalon);? Neurohypophysis
Proencephalon*;*diencephalon derivative
Q0266:Name the primary vesicle the following structures are
derived from (proencephalon; mesencephalon; or
rhombencephalon);? Third ventricle
Proencephalon
Q0267:Name the primary vesicle the following structures are
derived from (proencephalon; mesencephalon; or
rhombencephalon);? Hypothalamus
Proencephalon*;*diencephalon derivative
Q0268:Name the primary vesicle the following structures are
derived from (proencephalon; mesencephalon; or
rhombencephalon);? Lateral ventricles
Proencephalon
Q0269:What malignant tumor of the trophoblast causes high
levels of hCG and may occur after a hydatidiform mole;
abortion; or normal pregnancy?
Gestational trophoblastic neoplasia (GTN or
choriocarcinoma)
Q0270:What syndrome is due to a deficiency of surfactant?
Respiratory distress syndrome; treatment with cortisol and
thyroxine can increase production of surfactant.
Q0271:How many oogonia are present at birth?
None; they are not formed until a girl reaches puberty.
Q0272:What right-to-left shunt occurs when the aorta opens
into the right ventricle and the pulmonary trunk opens into
the left ventricle?
Transposition of the great vessels arises from a failure of the
aorticopulmonary septum to grow in a spiral.
Q0273:What are the adult remnants of the following
structures?;? Left umbilical vein
Ligament teres
Q0274:What are the adult remnants of the following
structures?;? Foramen ovale
Fossa ovale
Q0275:What are the adult remnants of the following
structures?;? Right and left umbilical arteries
Medial umbilical ligaments
Q0276:What are the adult remnants of the following
structures?;? Ductus arteriosus
Ligamentum arteriosum
Q0277:What are the adult remnants of the following
structures?;? Ductus venosus
Ligamentum venosum
Q0278:Mandibular hypoplasia; down-slanted palpebral
fissures; colobomas; malformed ears; and zygomatic
hypoplasia are commonly seen in what pharyngeal arch 1
abnormality?
Treacher Collins syndrome
Q0279:What is the tetrad of tetralogy of Fallot?
SHIP: Shifting of the aorta; Hypertrophy of the right
ventricle; Interventricular septal defect; Pulmonary stenosis
Q0280:What is the term for the external urethra opening onto
the ventral surface of the penis?
Hypospadia
Q0281:What CN is associated with the;? First pharyngeal
arch?
CN V
Q0282:What CN is associated with the;? Second pharyngeal
arch?
CN VII
Q0283:What CN is associated with the;? Third pharyngeal
arch?
CN IX
Q0284:What CN is associated with the;? Fourth pharyngeal
arch?
CN X
Q0285:What CN is associated with the;? Fifth pharyngeal
arch?
None; it degenerates.
Q0286:What CN is associated with the;? Sixth pharyngeal
arch?
CN X
Q0287:What disease results in a failure of neural crest cells to
migrate to the myenteric plexus of the sigmoid colon and
rectum?
Hirschsprung's disease (colonic gangliosus)
Q0288:What immunologic syndrome is due to a pharyngeal
pouch 3 and 4 failure?
DiGeorge's syndrome
Q0289:What embryonic structure; around day 19; tells the
ectoderm above it to differentiate into neural tissue?
The notochord
Q0290:What is the term for failure of the testes to descend
into the scrotum?
Cryptorchidism; normally the testes descend into the scrotum
within 3 months of birth.
Q0291:Is a membranous septal defect more commonly
interventricular or interatrial?
Membranous septal defects are interventricular; a persistent
patent ovale results in an interatrial septal defect.
Q0292:What pharyngeal pouch and groove persist when a
pharyngeal fistula is formed?
The second pharyngeal pouch and groove
Q0293:How early can a pregnancy be detected by hCG
assays in the blood? In urine?
hCG can be detected in the blood by day 8 and in the urine by
day 10.
Q0294:From what pharyngeal pouch is the following
structure derived?;? Middle ear
First;M PITS for pharyngeal pouch derivatives
Q0295:From what pharyngeal pouch is the following
structure derived?;? Superior parathyroid gland and
ultimobranchial body of the thyroid
Fourth;M PITS for pharyngeal pouch derivatives
Q0296:From what pharyngeal pouch is the following
structure derived?;? Inferior parathyroid gland and thymus
Third;M PITS for pharyngeal pouch derivatives
Q0297:From what pharyngeal pouch is the following
structure derived?;? Palatine tonsil
Second;M PITS for pharyngeal pouch derivatives
Q0298:What is the term for the external urethra opening onto
the dorsal surface of the penis?
Epispadia
Q0299:True or false? In females; meiosis II is incomplete
unless fertilization takes place.
True. The elimination of the unfertilized egg is menses.
Q0300:What adult structures are derived from preotic
somites?
Muscles of the internal eye
Q0301:What disorder is associated with jaundice; white
stools; and dark urine due to biliary duct occlusion secondary
to incomplete recanalization?
Extrahepatic biliary atresia
Q0302:What hormone; produced by the syncytiotrophoblast;
stimulates the production of progesterone by the corpus
luteum?
hCG
Q0303:How many mature sperm are produced by one type B
spermatogonium?
Four
Q0304:All primary oocytes in females are formed by what
age?
They are all formed by the fifth month of fetal life.
Q0305:From what embryonic structure are the following
structures derived?;? The ascending aorta and the pulmonary
trunk
Truncus arteriosus
Q0306:From what embryonic structure are the following
structures derived?;? The sinus venarum; coronary sinus; and
the oblique vein of the left atrium
Sinus venosus
Q0307:From what embryonic structure are the following
structures derived?;? The right and left ventricles
Primitive ventricle
Q0308:From what embryonic structure are the following
structures derived?;? The aortic vestibule and the conus
arteriosus
Bulbus cordis
Q0309:From what embryonic structure are the following
structures derived?;? The right and left atria
Primitive atrium
Q0310:After a longstanding left-to-right shunt reverses;
causing cyanosis; and becomes a right-to-left shunt; what is it
termed?
Eisenmenger's syndrome
Q0311:True or false? The thyroid gland is an embryologic
foregut derivative.
True. The thyroid gland; the lungs; and the pharyngeal
pouches are foregut derivatives that are not a component of
the gastrointestinal system.
Q0312:What embryonic structure forms the following adult
structures?;? Collecting ducts; calyces; renal pelvis; and ureter
Mesonephric duct (ureteric bud)
Q0313:What embryonic structure forms the following adult
structures?;? Urinary bladder and urethra
Urogenital sinus
Q0314:What embryonic structure forms the following adult
structures?;? External genitalia
Phallus; urogenital folds; and labioscrotal swellings
Q0315:What embryonic structure forms the following adult
structures?;? Nephrons; kidney
Metanephros
Q0316:What embryonic structure forms the following adult
structures?;? Median umbilical ligament
Urachus
Q0317:True or false? The epithelial lining of the urinary
bladder and the urethra are embryologic hindgut derivatives.
True
Q0318:Name the four ventral mesentery derivatives.
1. The lesser omentum (consisting of the hepatoduodenal and
hepatogastric ligaments) ;2. Falciform ligament ;3. Coronary
ligament of the liver ;4. Triangular ligament of the liver ;;Liver
is ventral; all other ligaments are dorsal mesentery derivatives.
Q0319:Projectile nonbilious vomiting and a small knot at the
right costal margin (olive sign) are hallmarks of what
embryonic disorder?
Hypertrophic pyloric stenosis due to hypertrophy of the
muscularis externa; resulting in a narrowed pyloric outlet
Q0320:The separation of 46 homologous chromosomes
without splitting of the centromeres occurs during what phase
of meiosis?
Meiosis I; disjunction with centromere splitting occurs during
meiosis II.
Q0321:Blood and its vessels form during what embryonic
week?
Third week; they are derived from the wall of the yolk sac.
Q0322:What embryonic structure forms the adult female
structures?;? Glans clitoris; corpus cavernosus; and
spongiosum
Phallus
Q0323:What embryonic structure forms the adult female
structures?;? Gartner's duct
Mesonephric duct
Q0324:What embryonic structure forms the adult female
structures?;? Ovary; follicles; rete ovarii
Gonads
Q0325:What embryonic structure forms the adult female
structures?;? Uterine tube; uterus; cervix; and upper third of
the vagina
Paramesonephric ducts
Q0326:What embryonic structure forms the adult female
structures?;? Labia majora
Labioscrotal swelling
Q0327:What embryonic structure forms the adult female
structures?;? Labia minora
Urogenital folds
Q0328:What embryonic structure forms the adult female
structures?;? Ovarian and round ligaments
Gubernaculum
Q0329:What embryonic structure forms the adult female
structures?;? Urinary bladder; urethra; greater vestibular
glands; vagina
Urogenital sinus
Q0330:What direction does the primitive gut rotate? What is
its axis of rotation?
The gut rotates clockwise around the superior mesenteric
artery.
Q0331:What syndrome occurs when a 46XY fetus develops
testes and female external genitalia?
Testicular feminization syndrome (Male who looks like a
female)
Q0332:Preeclampsia in the first trimester; hCG levels above
100; 00 mIU/mL; and an enlarged bleeding uterus are clinical
signs of what?
Hydatidiform mole
Q0333:True or false? The foramen ovale closes just prior to
birth.
False. It closes just after birth because the change in
pulmonary circulation causes increased left atrial pressure.
Q0334:At ovulation; in what stage of meiosis II is the
secondary oocyte arrested?
Metaphase II
Q0335:What is the name for failed recanalization of the
duodenum resulting in polyhydramnios; bile-containing
vomitus; and a distended stomach?
Duodenal atresia
Q0336:What remains patent in a hydrocele of the testis;
allowing peritoneal fluid to form into a cyst?
A patent processus vaginalis
Q0337:True or false? The respiratory system is derived from
the ventral wall of the foregut.
True. The laryngotracheal (respiratory) diverticulum is
divided from the foregut by the tracheoesophageal septum;;---
--------------------------------------------------------------------------
---
Q0338:What is the name for failure of the allantois to close;
resulting in a urachal fistula or sinus?
Patent urachus
Q0339:What structure is derived from the prochordal plate?
The mouth
Q0340:What is the only organ supplied by the foregut artery
that is of mesodermal origin?
Spleen
Q0341:What tumor is derived from primitive streak remnants
and often contains bone; hair; or other tissue types?
Sacrococcygeal teratoma
Q0342:What two pathologic conditions occur when the gut
does not return to the embryo?
Omphalocele and gastroschisis
Q0343:True or false? For implantation to occur the zona
pellucida must degenerate.
True. Remember; it degenerates 4 to 5 days post fertilization;
and implantation occurs 7 days post fertilization!
Q0344:What results when the maxillary prominence fails to
fuse with the medial nasal prominence?
Cleft lip
Q0345:What is the direction of growth for the primitive
streak; caudal to rostral or rostral to caudal?
The primitive streak grows caudal to rostral.
Q0346:During what embryonic week do somites begin to
form?
Third week
Q0347:In men; at what embryonic week do the primordial
germ cells migrate to the indifferent gonad?
Week four; and they remain dormant there until puberty.
Q0348:What embryonic week sees the formation of the
notochord and the neural tube?
Third week
Q0349:What right-to-left shunt occurs when only one vessel
receives blood from both the right and left ventricles?
Persistent truncus arteriosus
Q0350:What three embryonic cell layers form the chorion?
1. Cytotrophoblast ;2. Syncytiotrophoblast ;3.
Extraembryonic mesoderm
Q0351:Where are the preganglionic neuron cell bodies; the
CNS or the PNS?
They are in the grey matter of the CNS.
Q0352:Which three CNs send sensory information to the
solitary nucleus?
CN VII; IX; and X; taste and general sensation for the tongue
is sent to the solitary nucleus.
Q0353:What syndrome is associated with the following
brainstem lesions?;? Vertebral artery or anterior spinal artery
occlusion; resulting in contralateral corticospinal tract and
medial lemniscus tract deficits and an ipsilateral CN XII lesion
Medial medullary syndrome
Q0354:What syndrome is associated with the following
brainstem lesions?;? Contralateral corticospinal and medial
lemniscus tract deficits and an ipsilatera medial strabismus
secondary to a lesion in CN VI
Medial pontine syndrome
Q0355:What syndrome is associated with the following
brainstem lesions?;? Slow-growing acoustic neuroma
producing CN VII deficiencies
Pontocerebellar angle syndrome
Q0356:What syndrome is associated with the following
brainstem lesions?;? Occlusion of the PICA; resulting in
ipsilateral limb ataxia; ipsilateral facial pain and temperature
loss; contralateral pain and body temperature loss; ipsilateral
Horner's syndrome; and ipsilateral paralysis of the vocal
cords; palate droop; dysphagia; nystagmus; vomiting; and
vertigo
Lateral medullary (Wallenberg's) syndrome
Q0357:What syndrome is associated with the following
brainstem lesions?;? AICA or superior cerebellar artery
occlusion; resulting in ipsilateral limb ataxia; ipsilateral facial
pain and temperature loss; contralateral loss of pain and
temperature to the body; ipsilateral Horner's syndrome;
ipsilateral facial paralysis; and hearing loss
Lateral pontine syndrome
Q0358:What syndrome is associated with the following
brainstem lesions?;? Posterior cerebral artery occlusion
resulting in a contralateral corticospinal tract signs;
contralateral corticobulbar signs to the lower face; and
ipsilateral CN III palsy
Medial midbrain (Weber's) syndrome
Q0359:What CNs are affected if there is a lesion in;? The
midbrain?
CN III and IV
Q0360:What CNs are affected if there is a lesion in;? The
upper medulla?
CN IX; X; and XII
Q0361:What CNs are affected if there is a lesion in;?
Pontomedullary junction?
CN VI; VII; and VIII
Q0362:What CNs are affected if there is a lesion in;? The
upper pons?
CN V
Q0363:What is the only CN nucleus found in the cervical
spinal cord?
Accessory nucleus
Q0364:What component of the trigeminal nuclei;? Supplies
the muscles of mastication?
Motor nucleus of CN V
Q0365:What component of the trigeminal nuclei;? Receives
sensory input (all but pain and temperature) from the face;
scalp; dura; and the oral and nasal cavities?
Spinal trigeminal nucleus
Q0366:What component of the trigeminal nuclei;? Forms the
sensory component of the jaw jerk reflex?
Mesencephalic nucleus
Q0367:What deep cerebellar nuclei receive Purkinje cell
projections in;? The flocculonodular lobe?
The lateral vestibular nucleus
Q0368:What deep cerebellar nuclei receive Purkinje cell
projections in;? The vermis?
The fastigial nucleus
Q0369:What deep cerebellar nuclei receive Purkinje cell
projections in;? The lateral cerebellar hemispheres?
The interposed nucleus
Q0370:What deep cerebellar nuclei receive Purkinje cell
projections in;? The intermediate hemispheres?
The dentate nucleus
Q0371:What is the only excitatory neuron in the cerebellar
cortex; and what is its neurotransmitter?
The granule cell is the only excitatory neuron in the cerebellar
cortex; and it uses glutamate as its neurotransmitter. All the
other cells in the cerebellum are inhibitory neurons; and they
use GABA as their neurotransmitter.
Q0372:What three CNs are associated with conjugate eye
movements?
CN III; IV; and VI
Q0373:What is the term to describe the soft; flabby feel and
diminished reflexes seen in patients with acute cerebellar
injury to the deep cerebellar nuclei?
Hypotonia (rag doll appearance)
Q0374:What bedside test is used to differentiate a dorsal
column lesion from a lesion in the vermis of the cerebral
cortex?
The Romberg sign is present if the patient sways or loses
balance when standing with eyes open. In a dorsal column
lesion; patients sway with eyes closed. (Don't forget this
one.)
Q0375:Which one of the cerebellar peduncles is mainly
responsible for outgoing (efferent) information?
Superior cerebellar peduncle; the inferior and the middle
consist mainly of incoming (afferent) tracts and fibers.
Q0376:What tract carries unconscious proprioceptive
information from the Golgi tendon organs and muscle spindles
to the cerebellum; helping monitor and modulate muscle
movements?
Lower extremity and lower trunk information travels in the
dorsal spinocerebellar tract. The upper trunk and extremity
information travels in the cuneocerebellar tract.
(Cuneocerebellar and fasciculus cuneatus both apply to upper
extremities.)
Q0377:What reflex; seen in lesions of the corticospinal tract;
is an extension of the great toe with fanning the of remaining
toes?
The Babinski reflex is present in UMN lesions. Muscle
atrophy due to disuse; hyperreflexia; spastic paralysis;
increased muscle tone; and weakness are commonly seen in
UMN lesions.
Q0378:What is the triad of Horner's syndrome?
Ptosis (eyelid drooping); miosis (pupillary constriction); and
anhydrosis (lack of sweating) occur when the preganglionic
sympathetic fibers from T1-to T4 are obstructed.
Q0379:What component of the inner ear;? Contains
perilymph and responds to angular acceleration and
deceleration of the head?
Semicircular canal
Q0380:What component of the inner ear;? Contains
endolymph and responds to head turning and movement?
Semicircular duct
Q0381:What component of the inner ear;? Contains
endolymph and gravity receptors monitoring linear
acceleration and deceleration of the head; noting changes in
head position?
Utricle and saccule
Q0382:What is the name of demyelination of the corticospinal
tract and the dorsal column in the spinal cord due most
commonly to a vitamin B12 deficiency?
Subacute combined degeneration; which is bilateral below the
level of the lesion.
Q0383:What encephalopathy causes ocular palsies;
confusion; and gait abnormalities related to a lesion in the
mammillary bodies and/or the dorsomedial nuclei of the
thalamus?
Wernicke's encephalopathy
Q0384:Which thalamic nucleus receives auditory input from
the inferior colliculus?
MGB
Q0385:Where are the postganglionic neuron cell bodies; the
CNS or the PNS?
They are in ganglia in the PNS.
Q0386:What disease is a cavitation of the spinal cord causing
bilateral loss of pain and temperature at the level of the
lesion?
Syringomyelia
Q0387:What nucleus of the hypothalamus receives visual
input from the retina and helps set the circadian rhythm?
Suprachiasmatic nucleus
Q0388:Are white rami preganglionic or postganglionic fibers?
White rami are preganglionic fibers; whereas grey rami are
postganglionic fibers.
Q0389:What area of the hypothalamus is responsible for
recognizing a decrease in body temperature and mediates the
response to conserve heat?
Posterior hypothalamic zones; lesions here result in
poikilothermy (environmental control of one's body
temperature).
Q0390:What CN transmits sensory information from the
cornea?
CN V1; the occulomotor division of the trigeminal nerve; is
the sensory component of the corneal reflex.
Q0391:What preganglionic sympathetic fibers are responsible
for innervating the smooth muscle and glands of the pelvis and
the hindgut?
Lumbar splanchnics
Q0392:Where are the cell bodies for the DCML and
spinothalamic sensory systems?
The first sensory neuron is in the dorsal root ganglia. It carries
ascending sensory information in the dorsal root of a spinal
nerve; eventually synapsing with second sensory neuron. In
the brainstem (DCML) and the spinal cord (spinothalamic)
the second neuron cell body sends its axons to synapse in the
thalamus. The third sensory neuron cell body is a thalamic
nuclei that sends its fibers to the primary somatosensory
cortex.
Q0393:What term describes the reflex that increases the
curvature of the lens; allowing near vision?
Accommodation
Q0394:What CN carries preganglionic parasympathetic fibers
that innervate the viscera of the neck; thorax; foregut; and
midgut?
CN X (Remember; the vagus nerve supplies the
parasympathetic information from the tip of the pharynx to
the end of the midgut and all between.)
Q0395:What area of the hypothalamus is responsible for
recognizing an increase in body temperature and mediates the
response to dissipate heat?
Anterior hypothalamic zone; lesions here result in
hyperthermia.
Q0396:What excitatory fibers arise from the inferior olivary
nuclei on the contralateral side of the body?
Climbing fibers; they are monosynaptic input on Purkinje
cells. Mossy fibers; also excitatory; are axons of all other
sources and synapse on granule cells.
Q0397:What four CN carry preganglionic parasympathetic
fibers?
CN III; VII; IX; and X
Q0398:Name the form of spina bifida;? Meninges and spinal
cord project through a vertebral defect
Meningomyelocele;All except occulta cause elevated-
fetoprotein levels.
Q0399:Name the form of spina bifida;? Meninges project
through a vertebral defect
Meningocele;All except occulta cause elevated-fetoprotein
levels.
Q0400:Name the form of spina bifida;? An open neural tube
lying on the surface of the back
Myeloschisis;All except occulta cause elevated-fetoprotein
levels.
Q0401:Name the form of spina bifida;? Defect in the vertebral
arch
Occulta;All except occulta cause elevated-fetoprotein levels.
Q0402:Name the thalamic nucleus based on its input and
output;? Input from the optic tract; output projects to the
primary visual cortex of the occipital lobe
LGB (think EYES)
Q0403:Name the thalamic nucleus based on its input and
output;? Input from the trigeminal pathways; output to
primary somatosensory cortex of the parietal lobe
Ventral posteromedial nucleus
Q0404:Name the thalamic nucleus based on its input and
output;? Input from globus pallidus and the cerebellum;
output to the primary motor cortex
Ventral lateral nucleus
Q0405:Name the thalamic nucleus based on its input and
output;? Input from medial lemniscus and the spinocerebellar
tracts; output to the primary somatosensory cortex
Ventral posterolateral nucleus
Q0406:Name the thalamic nucleus based on its input and
output;? Input from globus pallidus and substantia nigra;
output to primary motor cortex
Ventral anterior nucleus
Q0407:Name the thalamic nucleus based on its input and
output;? Input from the amygdala; prefrontal cortex; and
temporal lobe; output to the prefrontal lobe and the cingulated
gyrus
Medial nuclear group (limbic system)
Q0408:Name the thalamic nucleus based on its input and
output;? Input from inferior colliculus; output to primary
auditory cortex
MGB (think EARS)
Q0409:Name the thalamic nucleus based on its input and
output;? Input from the mammillary bodies via the
mammillothalamic tract and the cingulated gyrus; output to
the cingulated gyrus via the anterior limb of the internal
capsule
Anterior nuclear group (Papez circuit of the limbic system)
Q0410:What is the name of a thin brown ring around the outer
edge of the cornea; seen in Wilson's disease?
Kayser-Fleischer ring
Q0411:What do UMNs innervate?
They innervate LMNs.
Q0412:What area of the brain serves as the major sensory
relay center for visual; auditory; gustatory; and tactile
information destined for the cerebral cortex; cerebellum; or
basal ganglia?
The thalamus (I like to think of the thalamus as the executive
secretary for the cerebral cortex. All information destined for
the cortex has to go through the thalamus.)
Q0413:Which of the colliculi help direct the movement of
both eyes in a gaze?
Superior colliculus (Remember S for Superior and Sight). The
inferior colliculus processes auditory information from both
ears.
Q0414:How do the corticobulbar fibres of CN VII differ from
the rest of the CNs?
Normally corticobulbar fiber innervation of the CNs is
bilateral (the LMN receives information from both the left and
right cerebral cortex); but with CN VII the LMN of the upper
face receives bilateral input but the lower facial LMNs receive
only contralateral input;;---------------------------------------------
-----------------------------------
Q0415:What syndrome is described by a lesion in the angular
gyrus (area 39) resulting in alexia; agraphia; acalculia; finger
agnosia; and right-left disorientation?
Gerstmann's syndrome; spoken language is usually
understood.
Q0416:How many pairs of spinal nerves are associated with;?
Cervical vertebrae?
Eight pairs through seven cervical vertebrae;Totaling 31 pairs
of spinal nerves.
Q0417:How many pairs of spinal nerves are associated with;?
Thoracic vertebrae?
Twelve pairs through twelve thoracic vertebrae;Totaling 31
pairs of spinal nerves.
Q0418:How many pairs of spinal nerves are associated with;?
Lumbar vertebrae?
Five pairs through five lumbar vertebrae;Totaling 31 pairs of
spinal nerves.
Q0419:How many pairs of spinal nerves are associated with;?
Sacral vertebrae?
Five pairs through five sacral vertebrae;Totaling 31 pairs of
spinal nerves.
Q0420:How many pairs of spinal nerves are associated with;?
Coccygeal vertebrae?
One pair with three to five coccygeal vertebrae;Totaling 31
pairs of spinal nerves.
Q0421:What are the three sites where CSF can leave the
ventricles and enter the subarachnoid space? (Name the lateral
and the medial foramina.)
Two Lateral foramina of Luschka and 1 Medial foramen of
Monroe (L for Lateral and M for Medial)
Q0422:What CNs arise from;? The midbrain?
CN III and IV
Q0423:What CNs arise from;? The pons?
CN V; VI; VII; and VIII
Q0424:What CNs arise from;? The medulla?
CN IX; X; and XII;CN XI arises from the cervical spinal cord.
Q0425:What disconnect syndrome results from a lesion in the
corpus callosum secondary to an infarct in the anterior
cerebral artery; so that the person can comprehend the
command but not execute it?
Transcortical apraxia; Wernicke's area of the left hemisphere
cannot communicate with the right primary motor cortex
because of the lesion in the corpus callosum.
Q0426:True or false? Glucose readily diffuses across the
blood-brain barrier.
False. Water readily diffuses across the blood-brain barrier;
but glucose requires carrier-mediated transport.
Q0427:What encapsulated group of nerve endings seen at the
muscle-tendon junction responds to an increase in tension
generated in that muscle? (This is dropping a box that is too
heavy to carry.)
Golgi tendon organs are stimulated by Ib afferent neurons in
response to an increase in force or tension. The inverse muscle
reflex protects muscle from being torn; it limits the tension on
the muscle.
Q0428:What chromosome 4; AD disorder is a degeneration of
GABA neurons in the striatum of the indirect pathway of the
basal ganglia?
Huntington's chorea; patients have chorea; athetoid
movements; progressive dementia; and behavioral problems.
Q0429:What syndrome is described as bilateral lesions of the
amygdala and the hippocampus resulting in placidity;
anterograde amnesia; oral exploratory behavior;
hypersexuality; and psychic blindness?
Klüver-Bucy syndrome
Q0430:By asking a patient to close the eyes while standing
with feet together; what two pathways are you eliminating
from proprioception?
When a patient closes the eyes while standing with feet
together; the visual and cerebellar components of
proprioception are removed; so you are testing the dorsal
columns. Swaying with eyes closed is a positive Romberg's
sign indicating a lesion in the dorsal columns;The cold water
caloric test mimics a brainstem lesion by inhibiting the normal
reflex response;(COWS: Cold Opposite Warm Same)
Q0431:What is the name of bilateral flaccid paralysis;
hyporeflexia; and hypotonia due to a viral infection of the
ventral horn of the spinal cord?
Poliomyelitis; it is a bilateral LMN lesion.
Q0432:What branch off the vertebral artery supplies;? The
ventrolateral two-thirds of the cervical spinal cord and the
ventrolateral part of the medulla?
Anterior spinal artery
Q0433:What branch off the vertebral artery supplies;? The
cerebellum and the dorsolateral part of the medulla?
PICA
Q0434:What syndrome causes inability to concentrate; easy
distractibility; apathy; and regression to an infantile suckling
or grasping reflex?
Frontal lobe syndrome (lesion in the prefrontal cortex)
Q0435:True or false? The presence of PMNs in the CSF is
always abnormal.
True. Although the CSF normally contains 0 to 4
lymphocytes or monocytes; the presence of PMNs is always
considered abnormal.
Q0436:What cells lining the ventricles have cilia on their
luminal surface to move CSF?
Ependymal cells
Q0437:What is the most common site for an aneurysm in
cerebral circulation?
The junction where the anterior communicating and anterior
cerebral arteries join. As the aneurysm expands; it compresses
the fibers from the upper temporal fields of the optic chiasm;
producing bitemporal inferior quadrantanopia
Q0438:What fissure of the cerebral cortex runs perpendicular
to the lateral fissure and separates the frontal and the parietal
lobes?
Central sulcus (sulcus of Rolando)
Q0439:What is the name of violent projectile movements of a
limb resulting from a lesion in the subthalamic nuclei of the
basal ganglia?
Hemiballismus
Q0440:What is the term for the type of pupil seen in
neurosyphilis; and what ocular reflexes are lost?
Argyll Robertson pupils accompany a loss of both direct and
consensual light reflexes; but the accommodation-convergence
reaction remains intact. It can also be seen in patients with
pineal tumors or multiple sclerosis.
Q0441:True or false? Intrafusal fibers form muscle spindles.
True. Muscle spindles are modified skeletal muscle fibers.
They are the sensory component of the stretch reflexes.
Q0442:What Brodmann area is associated with;? Broca's area?
Areas 44 and 45
Q0443:What Brodmann area is associated with;? Primary
auditory cortex?
Areas 41 and 42
Q0444:What Brodmann area is associated with;? Primary
somatosensory cortex?
Areas 1; 2; and 3
Q0445:What Brodmann area is associated with;?
Somatosensory association cortex?
Areas 5 and 7
Q0446:What Brodmann area is associated with;? Primary
motor cortex?
Area 4
Q0447:What Brodmann area is associated with;? Premotor
cortex?
Area 6
Q0448:What Brodmann area is associated with;? Visual
association cortex?
Areas 18 and 19
Q0449:What Brodmann area is associated with;? Frontal eye
fields?
Area 8
Q0450:What Brodmann area is associated with;? Primary
visual cortex?
Area 17
Q0451:What Brodmann area is associated with;? Wernicke's
area?
Area 22 and occasionally 39 and 40
Q0452:What is the fluid of the posterior compartment of the
eye?
Vitreous humor
Q0453:What aphasia produces a nonfluent pattern of speech
with the abilty to understand written and spoken language
seen in lesions in the dominant hemisphere?
Expressive aphasia
Q0454:In a topographical arrangement of the cerebellar
homunculus map; what area or lobe;? Controls the axial and
proximal musculature of the limbs?
The vermis
Q0455:In a topographical arrangement of the cerebellar
homunculus map; what area or lobe;? Is involved in motor
planning?
Lateral part of the hemispheres
Q0456:In a topographical arrangement of the cerebellar
homunculus map; what area or lobe;? Controls balance and
eye movements?
Flocculonodular lobe (one of my favorite words in all of
medicine!)
Q0457:In a topographical arrangement of the cerebellar
homunculus map; what area or lobe;? Controls distal
musculature?
Intermediate part of the hemispheres
Q0458:What glial cell is derived from mesoderm and acts as a
scavenger; cleaning up cellular debris after injury?
Microglia (Microglia and mesoderm both begin with M)
Q0459:What direct-pathway basal ganglia disease is described
by masklike facies; stooped posture; cogwheel rigidity; pill-
rolling tremor at rest; and a gait characterized by shuffling and
chasing the center of gravity?
Parkinson's disease (I can't underestimate all of the
buzzwords in this question. Remember it.)
Q0460:What artery supplies most of the lateral surfaces of
the cerebral hemispheres?
Middle cerebral artery
Q0461:What hypothalamic nucleus is responsible for the
production of ADH?
Supraoptic nuclei; lesions here result in diabetes insipidus.
Q0462:True or false? High-frequency sound waves stimulate
hair cells at the base of the cochlea.
True. High-frequency sound waves stimulate the hair cells at
the base of the cochlea; whereas low-frequency sound waves
stimulate hair cells at the apex of the cochlea.
Q0463:What nucleus of the hypothalamus is the satiety
center; regulating food intake?
Ventromedial nucleus; lesions here result in obesity.
Q0464:What cells of the retina sees in color and needs bright
light to be activated?
Cones (C for color and cones)
Q0465:What cell's axons are the only ones that leave the
cerebellar cortex?
The Purkinje cell
Q0466:What splanchnic carries preganglionic
parasympathetic fibers that innervate the hindgut and the
pelvic viscera?
Pelvic splanchnics (They all begin with P.)
Q0467:Is nystagmus defined by the fast or slow component?
Nystagmus is named by the fast component; which is the
corrective attempt made by the cerebral cortex in response to
the initial slow phase.
Q0468:Name the ocular lesion; be specific;? Left optic nerve
lesion
Left eye anopsia (left nasal and temporal hemianopsia)
Q0469:Name the ocular lesion; be specific;? Right calcarine
cortex lesion
Left homonymous hemianopsia
Q0470:Name the ocular lesion; be specific;? A right LGB
lesion (in the thalamus)
Left homonymous hemianopsia
Q0471:Name the ocular lesion; be specific;? Optic chiasm
lesion
Bitemporal heteronymous hemianopsia
Q0472:Name the ocular lesion; be specific;? A right lateral
compression of the optic chiasm (as in aneurysms in the
internal carotid artery)
Right nasal hemianopsia
Q0473:Name the ocular lesion; be specific;? Left Meyer's
loop lesion of the optic radiations.
Left homonymous hemianopsia
Q0474:What is the function of the cerebellum?
Planning and fine-tuning of voluntary skeletal muscle
contractions. (Think coordination.) Remember; the function of
the basal ganglia is to initiate gross voluntary skeletal muscle
control.
Q0475:What is the name for inability to stop a movement at
the intended target?
Dysmetria; this is seen in a finger-to-nose test.
Q0476:If a lesion occurs before the onset of puberty and
arrests sexual development; what area of the hypothalamus is
affected?
Preoptic area of the hypothalamus; if the lesion occurs after
puberty; amenorrhea or impotence will be seen.
Q0477:What sulcus divides the occipital lobe horizontally
into a superior cuneus and inferior lingual gyrus?
Calcarine sulcus
Q0478:Do alpha-or gamma-motor neurons innervate
extrafusal muscle fibers?
alpha-Motor neurons innervate extrafusal muscle fibers (a
motor unit); whereas gamma-motor neurons innervate
intrafusal muscle fibers.
Q0479:Contracting both medial rectus muscles
simultaneously makes the images of near objects remain on
the same part of the retina. What term describes this process?
Convergence
Q0480:Will a unilateral lesion in the spinothalamic tract result
in a contralateral or ipsilateral loss of pain and temperature?
Contralateral. The spinothalamic tract enters the spinal cord
and immediately synapses in the dorsal horn; crosses over;
and ascends contralateral in the spinal cord; brainstem;
thalamus; and postcentral gyrus.
Q0481:What ganglion supplies the postganglionic
parasympathetic fibers to the ciliary muscles of the eye?
Ciliary ganglion
Q0482:In what tract does pain; temperature; and crude touch
sensory information ascend to the postcentral gyrus of the
parietal lobe?
Spinothalamic tract (anterolateral system)
Q0483:What CN nucleus receives auditory information from
both ears via the cochlear nuclei?
Superior olivary nucleus
Q0484:What parasympathetic nucleus is found on the floor of
the fourth ventricle and supplies preganglionic fibers
innervating the terminal ganglias of the thorax; foregut; and
midgut?
Dorsal motor nucleus of CN X
Q0485:What sensory system is affected in the late spinal cord
manifestation of syphilis?
Bilateral degeneration of the dorsal columns in the spinal cord
secondary to syphilis is known as tabes dorsalis. A high-step
gait is seen in patients with tabes dorsalis because of the
inability to feel the ground beneath their feet.
Q0486:What do LMNs innervate?
They innervate skeletal muscle.
Q0487:What tract carries the ipsilateral dorsal column fibers
from the lower limbs in the spinal cord?
The fasciculus gracilis (Graceful); which lies closest to the
midline of the spinal cord.
Q0488:True or false? CSF is a clear; hypertonic solution with
higher concentrations of K + and HCO3-; than the serum.
False. CSF is a clear isotonic solution with lower
concentrations of K+ and HCO3-. It does have higher
concentrations of Cl- and Mg2+.
Q0489:What type of fiber or fibers are carried in (answer
motor; sensory; or both);? Dorsal root?
Sensory
Q0490:What type of fiber or fibers are carried in (answer
motor; sensory; or both);? Dorsal rami?
Both
Q0491:What type of fiber or fibers are carried in (answer
motor; sensory; or both);? Ventral rami?
Both
Q0492:What type of fiber or fibers are carried in (answer
motor; sensory; or both);? Ventral root?
Motor
Q0493:What type of fiber or fibers are carried in (answer
motor; sensory; or both);? Dorsal root ganglion?
Sensory
Q0494:What type of fiber or fibers are carried in (answer
motor; sensory; or both);? Spinal nerve?
Both
Q0495:Describe the loss for each of the following in a
hemisection of the spinal cord. (Brown–Sáequard syndrome);?
Dorsal column tract?
Ipsilateral loss at and below the level of the lesion
Q0496:Describe the loss for each of the following in a
hemisection of the spinal cord. (Brown–Sáequard syndrome);?
Corticospinal tract?
Ipsilateral loss below the level of the lesion
Q0497:Describe the loss for each of the following in a
hemisection of the spinal cord. (Brown–Sáequard syndrome);?
LMN?
Ipsilateral flaccid paralysis
Q0498:Describe the loss for each of the following in a
hemisection of the spinal cord. (Brown–Sáequard syndrome);?
Spinothalamic tract?
Contralateral loss below and bilateral loss at the level of the
lesion
Q0499:What area of the brain acts as the center for ipsilateral
horizontal gaze?
PPRF
Q0500:What aphasia is seen as an inability to comprehend
spoken language and speaking in a word salad?
Receptive aphasia is due to a lesion in Brodmann areas 22; 39;
and 40; generally the patient is unaware of the deficit.
Q0501:What is the function of the basal ganglia?
Initiate and manage gross skeletal muscle movement control
Q0502:What artery is formed by the union of the two
vertebral arteries?
The basilar artery is formed at the pontomedullary junction.
Q0503:What disease is described by bilateral flaccid weakness
of the upper limbs (LMN) and bilateral spastic weakness of
the lower limbs (UMN) beginning at the cervical level of the
spinal cord and progressing up or down the cord?
Amyotrophic lateral sclerosis (Lou Gehrig's disease) is a
LMN lesion at the level of the lesion and UMN lesion below
the level of the lesion.
Q0504:Which dopamine receptor excites the direct pathway
of the basal ganglia?
D1 receptor; inhibition of the direct pathway occurs through
the D2 receptors.
Q0505:Does the direct or indirect basal ganglia pathway result
in a decreased level of cortical excitation?
Although both pathways are associated with disinhibition; the
indirect basal ganglia pathway is associated with a decreased
level of cortical excitation.
Q0506:What fissure of the cerebral cortex separates the
frontal and temporal lobes rostrally and partially separates
the parietal and temporal lobes?
Lateral fissure (fissure of Sylvius)
Q0507:What area of the brain acts as the center for
contralateral horizontal gaze?
Frontal eye field (Brodmann area 8)
Q0508:In an adult; where does the spinal cord terminate and
what is it called?
The conus medullaris terminates at the level of the second
lumbar vertebra.
Q0509:If a patient with a cerebellar lesion has nystagmus;
which way is the fast component directed; toward or away
from the lesion?
The fast component is directed toward the affected side of a
cerebellar lesion.
Q0510:What area of the limbic system is responsible for
attaching emotional significance to a stimulus?
Amygdala; it helps imprint an emotional response in memory.
Q0511:What is the name of the tremor that occurs during
movements and is absent while the person is at rest?
Intention tremor; it is a sign of cerebellar lesions. A tremor at
rest (i.e; pill rolling) is seen in basal ganglia lesions.
Q0512:What is the term for making up stories regarding past
experiences because of an inability to retrieve them?
Confabulation; it is commonly seen in Korsakoff's syndrome.
Q0513:What frontal lobe cortex is associated with organizing
and planning the intellectual and emotional aspect of
behavior?
Prefrontal cortex; it is in front of the premotor area.
Q0514:What is the largest nucleus in the midbrain?
The substantia nigra is the largest nucleus in the midbrain. It
contains melanin and uses GABA and dopamine as its
neurotransmitters.
Q0515:Where is the lesion that produces these symptoms
when a patient is asked to look to the left?;? Left eye can't
look to the left
Left abducens nerve (VI)
Q0516:Where is the lesion that produces these symptoms
when a patient is asked to look to the left?;? Right eye can't
look left; left eye nystagmus; and convergence is intact
Right medial longitudinal fasciculus
Q0517:Where is the lesion that produces these symptoms
when a patient is asked to look to the left?;Neither eye can
look left with a slow drift to the right
Left abducens nucleus or right cerebral cortex
Q0518:What area of the hypothalamus is the feeding center?
Lateral hypothalamic zone; lesions here result in aphagia.
(Notice the difference between the feeding center and the
satiety center; they are in different zones.)
Q0519:In what pathway of the basal ganglia do lesions result
in hyperactive cortex with hyperkinetic; chorea; athetosis;
tics; and dystonia?
Indirect pathway (Tourette syndrome for example)
Q0520:What happens to muscle tone and stretch reflexes
when there is a LMN lesion?
The hallmarks of LMN lesion injury are absent or decreased
reflexes; muscle fasciculations; decreased muscle tone; and
muscle atrophy What two areas of the skin do flaccid
paralysis). Don't forget; LMN lesions are ipsilateral at the
level of the lesion!
Q0521:In what pathway of the basal ganglia do lesions result
in an underactive cortex with hypokinetic; slow; or absent
spontaneous movement?
Direct pathway; a good example is Parkinson's disease.
Q0522:What sided muscle weakness is seen in an UMN
corticospinal tract injury above the pyramidal decussation?
Contralateral muscle weakness when above the decussation;
whereas an UMN injury below the pyramidal decussation
results in ipsilateral muscle weakness.
Q0523:What area of the retina consists of only cones and has
the greatest visual acuity?
Fovea
Q0524:What tract carries the ipsilateral dorsal column fibers
from the upper limbs in the spinal cord?
The fasciculus cuneatus
Q0525:What CNS demyelinating disease is characterized by
diplopia; ataxia; paresthesias; monocular blindness and
weakness; or spastic paresis?
Multiple sclerosis
Q0526:What part of the ANS (i.e; PNS or CNS) controls the
constriction of the pupil in response to light?
Parasympathetic
Q0527:With which CN are preganglionic parasympathetic
axons arising from the Edinger- Westphal nucleus associated?
CN III
Q0528:Ophthalmic artery is a branch of what artery?
Internal carotid artery
Q0529:What thalamic relay nucleus do the mammillary bodies
project to?
The anterior nucleus of the thalamus
Q0530:What cells contribute to the blood-brain barrier and
proliferate in response to CNS injury?
Astrocytes
Q0531:What causes slow writhing movements (athetosis)?
Hypermyelination of the corpus striatum and the thalamus
(seen in cerebral palsy)
Q0532:What area of the brain is responsible for emotion;
feeding; mating; attention; and memory?
The limbic system
Q0533:What is the name of the postganglionic
parasympathetic ganglion that innervates;? The papillary
sphincter and ciliary muscle of the eye?
Ciliary ganglion. (These fibers are carried in CN III.
Remember it like this:-ili-in ciliary ganglion looks like the III
of CN III.)
Q0534:What is the name of the postganglionic
parasympathetic ganglion that innervates;? The parotid gland?
The otic ganglion. (These fibers are carried in CN IX.
Remember it like this: the -oti-is in both otic ganglion and
parotid gland.)
Q0535:What is the name of the postganglionic
parasympathetic ganglion that innervates;? The
submandibular and sublingual glands?
The submandibular ganglion. (Submandibular ganglion
innervates the submandibular gland; easy enough.)
Q0536:What is the name of the postganglionic
parasympathetic ganglion that innervates;? The lacrimal gland
and oral and nasal mucosa?
Pterygopalatine ganglion (I remember this as the only ganglion
left.)
Q0537:What neuronal cell bodies are contained in the
intermediate zone of the spinal cord? (T1–L2)
Preganglionic sympathetic neurons
Q0538:What limb of the internal capsule is not supplied by
the middle cerebral artery?
Anterior limb of the internal capsule is supplied by the
anterior cerebral artery.
Q0539:What tract is responsible for voluntary refined
movements of distal extremities?
Corticospinal tract
Q0540:Craniopharyngiomas are remnants of what?
Rathke's pouch; they can result in compression of the optic
chiasm.
Q0541:Clarke's nucleus is the second ascending sensory
neuron of which spinocerebellar tract?
Dorsal spinocerebellar tract; the accessory cuneate nucleus is
the second nucleus for the cuneocerebellar tract.
Q0542:Name the three postganglionic sympathetic ganglia
that receive input from thoracic splanchnics.
Celiac; aorticorenal; and superior mesenteric ganglias.
(Remember all " Splanchnics" are Sympathetic except for the
Pelvic splanchnics; which are Preganglionic Parasympathetic
fibers.)
Q0543:What is the only CN to arise from the dorsal surface
of the midbrain?
CN IV
Q0544:What basic reflex regulates muscle tone by contracting
muscles in response to stretch of that muscle?
The myotatic reflex is responsible for the tension present in
all resting muscle.
Q0545:Where are the LMN cell bodies of the corticospinal
tract?
In the ventral horn of the spinal cord. UMN cell bodies are in
the precentral gyrus of the frontal lobe.
Q0546:What nucleus; found in the intermediate zone of the
spinal cord; sends unconscious proprioception to the
cerebellum?
Clarke's nucleus
Q0547:The vertebral artery is a branch of what artery?
The subclavian artery
Q0548:What muscle of the middle ear is innervated by the
mandibular division of CN V?
Tensor tympani
Q0549:The fibers of nucleus gracilis and nucleus cuneatus
cross at the medullary decussation and ascend contralateral to
what thalamic relay nucleus?
VPL nucleus sends its fibers to synapse in the postcentral
gyrus of the parietal lobe.
Q0550:What muscle of the middle ear is innervated by CN
VII?
The stapedius muscle
Q0551:What part of the inner ear contains the gravity
receptors for changes in the position of the head?
Saccule and utricle
Q0552:What nucleus supplies the preganglionic
parasympathetic fibers to the ciliary ganglion?
Edinger-Westphal nucleus (via CN III)
Q0553:What reticular nuclei synthesize serotonin from L-
tryptophan and plays a role in mood; aggression; and inducing
sleep?
The raphe nuclei
Q0554:Will a patient with a unilateral lesion in the cerebellum
fall toward or away from the affected side?
Patients with unilateral cerebellar lesions fall toward the side
of the lesion.
Q0555:A unilateral lesion in what nucleus will produce
ipsilateral paralysis of the soft palate?
Nucleus ambiguus; resulting in the uvula deviating away from
the side of the lesion.
Q0556:True or false? Neurons in the dorsal horn participate
in reflexes.
True. They are the sensory component of a spinal reflex.
Q0557:What ganglion receives preganglionic sympathetic
fibers from T1 to L1–2 and innervates smooth muscle; cardiac
muscle; glands; head; thoracic viscera; and blood vessels of the
body wall and limbs?
Sympathetic chain ganglion
Q0558:What preganglionic sympathetic fibers are responsible
for innervating the foregut and the midgut?
Thoracic splanchnic fibers
Q0559:Does light or darkness regulate the pineal gland?
Light regulates the activity of the pineal gland via the retinal–
suprachiasmatic– pineal pathway.
Q0560:Name the three hormones produced by pinealocytes.
Melatonin; serotonin; and CCK
Q0561:Is the pH of CSF acidotic; alkalotic; or neutral?
The pH of CSF is 7.33; acidotic.
Q0562:What ascending sensory system carries joint position;
vibratory and pressure sensation; and discriminative touch
from the trunk and limbs?
The DCML system. (Remember; everything but pain and
temperature.)
Q0563:What reflex enables the eyes to remain focused on a
target while the head is turning?
The vestibulo-ocular reflex
Q0564:What cells of the retina see in black and white and are
used for night vision?
Rods
Q0565:Name the muscle type based on these descriptions;?
Discontinuous voluntary contraction; multinuclear striated
unbranched fibers; actin and myosin overlapping for banding
pattern; triadic T tubules; troponin and desmin as Z disc
intermediate filament.
Skeletal muscles
Q0566:Name the muscle type based on these descriptions;?
Continuous involuntary contraction; uninuclear striated
branched fibers; actin and myosin overlapping for banding
pattern; dyadic T tubules; intercalated discs; troponin and
desmin as a Z disc intermediate filament.
Cardiac muscle
Q0567:Name the muscle type based on these descriptions;?
Involuntary contraction; uninuclear nonstriated fibers; actin
and myosin not forming banding pattern; lack of T tubules;
gap junctions; and calmodulin.
Smooth muscle
Q0568:What segment of the small intestine is associated with
Brunner's glands?
Duodenum
Q0569:Who is responsible for passing on mitochondrial DNA
genetic disorders?
Mitochondria-linked disorders are always inherited from the
mother.
Q0570:What part of a neuron receives information?
Dendrites receive information; whereas axons send
information.
Q0571:What type of collagen is associated with the basement
membrane?
Type IV collagen
Q0572:What is the epithelial lining of the prostatic portion of
the urethra?
Transitional epithelium. The distal portion of the penile
urethra is composed of stratified epithelium.
Q0573:What cell of the nephron is responsible for renin
production and secretion?
Juxtaglomerular (JG) cell
Q0574:What cell surface modification of ependymal cells and
respiratory epithelium has a 9 + 2 microtubular configuration
and movement as its function?
Cilia
Q0575:True or false? The following are functions of
hepatocytes: protein production; bile secretion; detoxification;
conjugation; and lipid storage.
True. (They are quite a busy bunch of cells!)
Q0576:What substance found in eosinophils is toxic to
parasitic worms?
Major basic protein
Q0577:After fertilization; what cells of the corpus luteum;?
Secrete progesterone?
Granulose cells secrete progesterone. After fertilization the
granulose cells form from follicular cells.
Q0578:After fertilization; what cells of the corpus luteum;?
Secrete estrogen?
Theca cells secrete estrogen. After fertilization the theca cells
form from the theca interna.
Q0579:What is the largest organ in the body?
Integument (skin and its derivatives)
Q0580:On what layer of the epidermis does all mitosis occur?
Malpighian layer (made up of the stratum basale and stratum
spinosum)
Q0581:What ribosomal subunit binds first to the mRNA
strand?
The small subunit (40S) binds first.
Q0582:What is the T-cell area of the spleen?
PALS
Q0583:What element is needed for the proper alignment of
tropocollagen molecules?
Copper (Cu+)
Q0584:What type of cell surface projection lies on the lateral
surface of cells closest to the apex and acts to seal off the
outside environment from the rest of the body?
Zonula occludens (tight junctions)
Q0585:What organelle is responsible for ribosomal RNA
synthesis?
Nucleolus. Ribosomal assembly also takes place in the
nucleolus.
Q0586:What sweat gland type is associated with odor
production and hair follicles and is found in the axilla?
APocrinE glands (APES is my memory aid) Axilla; Areola;
and Anus all begin with A. APES are hairy (associated with
hair follicles). They smell (odor production); and if confronted
by an APE; your Adrenergic nervous system would be firing
(innervation).
Q0587:What papillae send their senses via chorda tympani of
CN VII?
Fungiform papillae
Q0588:True or false? The portal tract of the liver lobule is the
first area to be oxygenated in the liver.
True. (Remember; blood flows from the portal tracts to the
central vein; so it is the first area to receive blood and
therefore oxygen.)
Q0589:Match the chromosome and haploid number with the
stage of sperm development; spermatid; spermatocyte
(primary or secondary); spermatogonia (type A or B);? 46/2n
(divide meiotically)
Spermatogonia (type B)
Q0590:Match the chromosome and haploid number with the
stage of sperm development; spermatid; spermatocyte
(primary or secondary); spermatogonia (type A or B);? 46/4n
Primary spermatocyte
Q0591:Match the chromosome and haploid number with the
stage of sperm development; spermatid; spermatocyte
(primary or secondary); spermatogonia (type A or B);? 23/1n
Spermatid
Q0592:Match the chromosome and haploid number with the
stage of sperm development; spermatid; spermatocyte
(primary or secondary); spermatogonia (type A or B);? 46/2n
(divide mitotically)
Spermatogonia (type B)
Q0593:Match the chromosome and haploid number with the
stage of sperm development; spermatid; spermatocyte
(primary or secondary); spermatogonia (type A or B);? 23/2n
Secondary spermatocyte
Q0594:What are the four functions of SER?
Steroid synthesis; drug detoxification; triglyceride resynthesis;
and Ca2+handling
Q0595:Which immunoglobulin is secreted by the plasma cells
in the gastrointestinal tract?
IgA
Q0596:What area of the lymph node is considered the
thymic-dependent area?
The inner cortex (paracortex) contains the T cells; so it is
considered the thymic-dependent area.
Q0597:What type of chromatin is transcriptionally inactive?
Heterochromatin; the light stuff in the nucleus on an electron
microscope image.
Q0598:Both submandibular and sublingual glands are
innervated by CN VII (facial) and produce mucous and serous
secretions. Which one mainly produces serous secretions?
Submandibular gland produces mainly serous and the
sublingual gland produces mainly mucous secretions.
Q0599:What is the only neuroglial cell of mesodermal origin?
Microglia. All others are neuroectodermal derivatives.
Q0600:Where is tropocollagen aggregated to form a collagen
fibril?
Outside the cell
Q0601:What are the four posttranslational modifications done
by the Golgi apparatus?
1. Phosphorylation of mannose (lysosomes only) ;2. Removal
of mannose residues ;3. Formation of glycosylate proteins ;4.
Phosphorylation of sulfate amino acids
Q0602:What is the epithelial cell lining the nasopharynx?
Stratified squamous nonkeratinized epithelium; which has cilia
that beat toward the oropharynx.
Q0603:What are the three epidermal derivatives?
1. Nails ;2. Hair ;3. Sweat glands (both apocrine and
sebaceous)
Q0604:What are the long microvilli in the inner ear and male
reproductive tract called?
Stereocilia
Q0605:True or false? The central vein of the liver lobule is the
first area affected during hypoxia.
True. Blood flows from the portal tracts (distal) to the central
vein (proximal); so it is the first area affected during hypoxia.
Q0606:What cell of the male reproductive system produces
testosterone?
Leydig cells produce testosterone. LH stimulates Leydig cells.
(Both start with L.)
Q0607:Myelin is produced by which cells in the PNS? In the
CNS?
In the PNS; myelin is produced by Schwann cells; in the CNS
by oligodendrocytes.
Q0608:What cell type of the epidermis functions as antigen-
presenting cells?
Langerhans cells (found in the stratum spinosum)
Q0609:What cell type is found in the peripheral white pulp
of the spleen?
B cells are mainly found in the peripheral white pulp and
germinal centers in the spleen.
Q0610:What area of the female reproductive tract is lined by
stratified squamous epithelium rich in |glycogen?
The vagina
Q0611:What encapsulated lymphoid organ is characterized by
presence of Hassall's corpuscles; and absence of germinal
centers and B cells?
Thymus gland. (Thymus gland is essential for T cell
maturation.)
Q0612:What cell transports IgA; is secreted by plasma cells;
and is in Peyer's patches to the gastrointestinal lumen?
M-cells
Q0613:What are the cells of the parathyroid gland that
produce PTH?
Chief cells
Q0614:What skin type on the palms and soles is
characterized by the absence of hair follicles and presence of
stratum lucidum?
Thick skin
Q0615:What is the name of hydrophilic pores that allow the
direct passage of ions and particles between two adjacent
cells?
Gap junctions
Q0616:What type of lysosome is formed when lysosome
fuses with a substrate for breakdown?
Secondary lysosome (think of the primary as inactive and
secondary as active)
Q0617:What cell membrane structure increases the surface
area of a cell and has actin randomly assorted within its
structure?
Microvillus
Q0618:What are the four components of the basement
membrane?
1. Laminin ;2. Heparan sulfate (heparitin sulfate) ;3.
Fibronectin ;4. Type IV collagen
Q0619:What organelle synthesizes proteins that are intended
to stay within the cell?
Free polysomes. Membrane-associated polysomes are the site
of protein synthesis destined to leave the cell.
Q0620:What cell type of the body or fundus of the stomach
secretes IF?
Parietal cells (Remember; they secrete HCl; too.)
Q0621:What cell type of the body or fundus of the stomach
secretes pepsinogen?
Chief cells
Q0622:What hormone; produced by the granulose cell;
stimulates the endometrium to enter the proliferative phase?
Estrogen; the first 14 days of the female reproductive cycle
mark the proliferative phase.
Q0623:What cells of the nephron function as sodium
concentration sensors of the tubular fluid?
Macula densa
Q0624:What type of chromatin is transcriptionally active?
Euchromatin; the dark stuff in the nucleus on an electron
microscope image.
Q0625:What cells of the thyroid gland secrete calcitonin?
Parafollicular C cells
Q0626:True or false? The nucleus is the site of transcription.
True. Transcription (conversion of DNA to RNA); as well as
replication; occurs in the nucleus.
Q0627:How many days after the LH surge is ovulation?
One day after the LH surge and 2 days after the estrogen
peak.
Q0628:In what layer of the epidermis is melanin transferred
from melanocytes to keratinocytes?
Stratum spinosum
Q0629:What cells of the epidermis; derived from the neural
crest; act as mechanoreceptors?
Merkel cells (Merkel's tactile cells)
Q0630:What substance do the JG cells of the kidney secrete
in response to low blood pressure?
Renin
Q0631:What is the rule of one-third regarding muscle type of
the esophagus?
Upper third skeletal muscle; middle third both skeletal and
smooth muscle; and lower third smooth muscle
Q0632:What papillae are responsible for sweet taste?
Circumvallate papillae
Q0633:What area of the lymph node contains germinal
centers?
The outer cortex contains most of the germinal centers and
therefore also most B cells.
Q0634:True or false? The gallbladder functions to produce
bile.
False. The gallbladder does not produce bile; but it
concentrates bile via active sodium transport; water follows
the sodium.
Q0635:True or false? Depolarization of the postsynaptic
membrane excites the neuron.
True. Hyperpolarization inhibits the postsynaptic membrane.
Q0636:In the alveoli; what cell type is;? for gas exchange?
Type I pneumocytes
Q0637:In the alveoli; what cell type is;? responsible for
producing surfactant?
Type II pneumocytes
Q0638:In the alveoli; what cell type is;? part of the
mononuclear phagocytic system?
Alveolar macrophages (dust cells)
Q0639:Which trophoblast layer of the placenta remains until
the end of pregnancy?
Syncytiotrophoblast. (The cytotrophoblast gets incorporated
into the syncytiotrophoblast.)
Q0640:What is the first epidermal layer without organelles
and nuclei
Stratum lucidum
Q0641:What area of the small intestine is characterized by
Peyer's patches?
Ileum
Q0642:What lymphoid organ has the following
characteristics: outer and inner cortical areas; encapsulation;
germinal centers; and high endothelial venules?
Lymph nodes
Q0643:What area of the nephron is sensitive to the effects of
ADH?
Collecting ducts; which make them readily permeable to water
reabsorption.
Q0644:What is the name of RER in neurons?
Nissl substances; there is a great deal of RER in neuron cell
bodies; indicating high protein synthesis.
Q0645:What hormone causes milk letdown?
Oxytocin
Q0646:What are the three reasons for the effectiveness of the
blood-brain barrier?
1. Tight junctions ;2. Capillaries that lack fenestration ;3.
Very selective pinocytosis by the capillaries
Q0647:What cell type of the epidermis originates from the
neural crest?
Melanocytes
Q0648:If no fertilization occurs; how many days after
ovulation does the corpus luteum begin to degenerate?
12 days after ovulation
Q0649:What area of the spleen consists of splenic cords of
Billroth and phagocytoses RBCs?
Red pulp (Remember; Red pulp and RBCs begin with R.)
Q0650:What is the name of the protein coat that surrounds
the nuclear envelope?
Vimentin
Q0651:What papillae are touch receptors on the tongue and
send their sensations via CN V3 (mandibular division)?
Filiform papillae
Q0652:What is the most superficial layer of the epidermis?
Stratum corneum (keratinized)
Q0653:What syndrome is characterized by dynein arm
abnormality resulting in chronic sinusitis; recurrent
pulmonary infections; and infertility?
Kartagener's syndrome (also known as immotile cilia
syndrome);;------------------------------------------------------------
--------------------
Q0654:What are the functions of the zonula occludens and the
zonula adherens
To provide attachment between contiguous cells and to
maintain a semipermeable barrier
Q0655:What is the name of the SER of striated muscle?
Sarcoplasmic reticulum
Q0656:Where do sperm go for maturation?
Ductus epididymis; which is lined by pseudostratified
epithelium with stereocilia.
Q0657:When is the first arrested stage of development in the
female reproductive cycle?
Prophase of meiosis I (between 12th and 22nd week in utero)
Q0658:What is the longest and most convoluted segment of
the nephron?
PCT
Q0659:What cells of the epidermis carry the pigment
melanin?
Keratinocytes; the most numerous cells in the epidermis;
carry melanin and produce keratin.
Q0660:What segment of the gastrointestinal tract lacks villi;
has crypts; and actively transports sodium out of its lumen?
Large intestine. Water is passively removed from the lumen.
Q0661:What two areas of the skin do not contain sebaceous
glands?
Palms and soles of the feet. Sebaceous glands are associated
with hair follicles; which are lacking on the palms and soles of
the feet.
Q0662:Which of the following is not part of the conducting
portion of the respiratory system: trachea; bronchi; alveoli; or
larynx?
Alveoli; they are part of the respiratory portion.
Q0663:Where are the enzymes for the ETC and oxidative
phosphorylation found?
The inner membrane of the mitochondria (cristae)
Q0664:What lymphoid organ is characterized by germinal
centers; plasma cells that secrete IgA; and no encapsulation?
Peyer's patch
Q0665:What generate anterograde transport of information in
a neuron?
Kinesins. Dynein generates retrograde transportation of
information.
Q0666:What is the only glycosaminoglycan (GAG) that binds
to the linker portion of the proteoglycan?
Hyaluronic acid (all sulfates bind to the core protein)
Q0667:What cell in bone is a part of the mononuclear
phagocytic system?
Osteoclasts
Q0668:What three factors do Sertoli cells produce for normal
male development?
Inhibin; müllerian-inhibiting factor; and androgen binding
protein
Q0669:What epidermal layer's function is to release lipids to
act as a sealant?
Stratum granulosum
Q0670:What does the tunica intima of arteries have that veins
do not?
An internal elastic lamina
Q0671:Do the duct or the acini cells of the pancreas secrete
HCO3-?
Duct cells secrete HCO3-; electrolytes; and water. The acini
secrete the enzymes necessary for carbohydrate; nucleic acid;
protein cleavage; and emulsification of fats.
Q0672:What cell of the duodenum contains high
concentrations of lysozymes and has phagocytic activity?
Paneth cells
Q0673:What maintains the osmotic gradient that is critical to
the concentrating ability of the kidney?
The venae recta maintain the gradient via countercurrent flow.
Q0674:Are the JG cells of the nephron a part of the afferent
or efferent arteriole?
Afferent arteriole
Q0675:What cell of the duodenum secretes CCK?
Enteroendocrine (EE) cells; they also secrete secretin.
Q0676:What are the proteoglycans of cartilage and bone?
Chondroitin sulfate and keratan sulfate
Q0677:What is the term for the first 3 to 5 days of the female
reproductive cycle?
Menses. (Ovulation occurs 14 days before the beginning of
menses.)
Q0678:What is the second arrested stage of development in
the female reproductive cycle?
Metaphase of meiosis II (in the oocyte of the graafian follicle)
Q0679:What ribosomal subunit sizes do eukaryotic cells
have?
60S and 40S. The large subunits (60S) are made in the
nucleolus and the small subunits (40S) are made in the
nucleus.
Q0680:What term describes how an action potential is
propagated along an axon?
Saltatory conduction
Q0681:What phase of the female reproductive cycle is 14
days long?
The secretory phase is progesterone-dependent and 14 days
long; whereas the length of the proliferative phase varies
Q0682:A single mRNA strand translated by a ribosome is
termed what?
Polysome. Ribosomes read from the 5' to the 3' end of the
mRNA.
Q0683:What cell is under control of FSH and testosterone;
secretes inhibin; MIF; and androgen-binding protein; and
phagocytizes the excess cytoplasm of the spermatid?
Sertoli cell
Q0684:What histone binds two nucleosomes together?
H1 histones
Q0685:What is the major inorganic component of bone?
Hydroxyapatite
Q0686:What cells of the adrenal gland are neural crest
derivatives?
Chromaffin cells (adrenal medulla)
Q0687:Where does Beta-oxidation of very long chain fatty
acids begin?
In the peroxisome until it is 10 carbons long; the rest is
completed in the mitochondria.
Q0688:What organelles make ATP; have their own dsDNA;
and can synthesize protein?
Mitochondria
Q0689:aortic arch found at what vertebral level?
T2 (pulsatile mass at T2-->aortic arch aneurysm)
Q0690:alar ligaments
limit excessive movement of atlanto-axial jount (turning head
"no")
Q0691:what happens if transverse (cruciform) ligament tears?
(due to trauma e.g. Jefferson fx; or RA); allows dens to move
within vertebral canal
Q0692:dislocations without fracture are found only within
what region of vertebral column?
cervical region b/c here the articular surfaces are inclined
horizontally
Q0693:"fish mouth vertebrae"
assoc'd with sickle cell anemia; central depressions seen
radiographically in vertebral body
Q0694:bifurcation of common carotid artery at what vertebral
level?
C4
Q0695:cricoid cartilage at what vertebral level?
C6
Q0696:teardrop fx caused by?
hyperflexion of cervical region; e.g. from diving into shallow
water; whiplash
Q0697:Jefferson fracture
caused by compression of cervical region (force applied to top
of head); fx of C1 at multiple sites; tear of transverse ligament
Q0698:hangman fracture
hyperextension of cervical region
Q0699:Chance fracture
thoracolumbar column trauma caused by hyperflexion of
thoracic or lumbar region (e.g. seat belt injury); usually does
not place spinal cord at risk
Q0700:dorsal intermediate septum of spinal cord; where?
only above T6 (separates gracilus and cuneatus)
Q0701:filum terminale is extension of what?
pia matter
Q0702:interal vertebral venous plexus where?
epidural space
Q0703:what happens if artery of Adamkiewicz ligated during
resection fo AAA?
may result in anterior spinal artery syndrome: paraplegia;
impotence; bladder/bowel dysfxn; loss of pain and T
sesnation
Q0704:dorsal primary ramus innervates what?
skin and deep muscles of back
Q0705:thumb dermatome
C6
Q0706:upper neck dermatome
C3
Q0707:ring and little fingers dermatome
C8
Q0708:medial surface of leg; big toe dermatome
L4
Q0709:lateral surface of leg dermatome
L5
Q0710:dorsum of foot dermatome
L5
Q0711:poster surface of lower limb dermatome
S1
Q0712:lateral surface of foot dermatome
S1
Q0713:little toe dermatome
S1
Q0714:genitalia and anal zone dermatome
S2-S5
Q0715:lumbar puncture needle passes through?
skn->superficial fascia-> supraspinous ligament->
interspinous ligament-> ligamentum flavum-> epidural space-
> dura mater-> arachnoid-> subarachnoid space
Q0716:how can breast cancer metastasize to brain?
intercostal vein-> external vertebral plexus-> internal vertebral
plexus-> cranial dural sinus
Q0717:what important structures cross rib 1?
subclavian a/v; brachial plexus
Q0718:aspiration of peanut when sitting/standing
right lower lobe; lower portion
Q0719:where does aspirated peanut go when supine?
right lower lobar bronchus; upper portion
Q0720:where does aspirated peanut go when lying on right
side?
right UPPER lobar bronchus; posterior portion
Q0721:where does aspirated peanut go when lying on left
side?
LEFT UPPER lobar bronchus; lingula
Q0722:4 stages of inflammatory response in lobar pneumonia
1. initial (acute congestion; intraalveolar fluid; few PMNs;
many bact.); 2. early consolidation/"red hepatization"
(extravasated RBCs); 3. late consolidation/"grey
hepatization"; 4. resolution
Q0723:middle cardiac vein follows what artery?
posterior interventricular artery
Q0724:small cardiac vein follows what artery?
right marginal artery
Q0725:cell bodies of preganglionic parasymp neurons that
decrease heart rate located where?
dorsal nucleus of vagus and nucleus ambiguus of medulla
Q0726:cancer of scrotum metastasizes where?
superficial inguinal nodes
Q0727:cancer of testes metastasizes where?
deep lumbar nodes near renal hilus (b/c of embyro origin!)
Q0728:name the hernia: protrudes medial to inferior epigastric
a/v; older males
direct inguinal hernia (thru Hesselbach triangle)
Q0729:name the hernia: protrudes lateral to inferior epigastric
a/v; medial and above to pubic tubercle
indirect hernia (more common than direct)
Q0730:name the hernia: protrudes below and lateral to pubic
tubercle
femoral hernia (more common in females; esp on right side)
Q0731:is pancreas intra- or retroperitoneal?
head; neck; and body--RETRO; tail--INTRA
Q0732:celiac trunk--at what vertebral level? major branches?
at T12; left gastric artery; splenic artery; common hepatic
artery
Q0733:superior mesenteric artery at what vertebral level?
L1
Q0734:inferior mesenteric artery at what vertebral level?
L3
Q0735:gastric ulcers most commonly where?
in body of stomach along lesser curvature above incisura
angularis
Q0736:significance of prepyloric vein
makrs gastroduodenal jxn
Q0737:who has longer vasa recta--jejunum or ileum?
jejunum
Q0738:what are Rokitansky-Aschoff sinuses?
early indicators of pathology changes in gallbladder; mucosa
of gallbladder penetrates deep into muscularis externa
Q0739:MIrizzi syndrome
large gallstone impacted in cystic duct extrinsically obstructs
nearby common hepatic duct
Q0740:embryo origin of uncinate process of pancreas?
ventral pancreatic bud
Q0741:embryo origin of pancreatic head?
ventral AND dorsal pancreatic buds
Q0742:embryo origin of neck; body; and tail of pancreas?
dorsal pancreatic bud
Q0743:lymph drainage of lower anal canal vs upper anal canal
lower--superficial inguinal; upper--deep nodes
Q0744:arterial supply of lower anal canal vs upper anal canal
lower--inferior rectal artery (branch of internal pudendal);
upper--superior rectal artery (br of inf mesenteric)
Q0745:venous drainage of lower anal canal vs upper anal canal
lower--to IVC; upper-too hepatic portal system
Q0746:contents of splenorenal ligament
terminal branches of splenic artery; tributaries of splenic vein;
tail of pancrease
Q0747:most common complication of splenectomy?
atelectasis
Q0748:if superior wall of bladder ruptured (e.g. compression
on full bladder) where would pee go?
peritoneal cavity
Q0749:if anterior wall of bladder ruptured; where would piss
go?
extraperitoneal extravasation of urine w/i retropubic space of
Retzius (e.g. fractured pelvis in car accident)
Q0750:where would piss go if urethra ruptured below
urogenital diaphragm
superficial perineal space
Q0751:Stage I RCC
confined to renal capsule
Q0752:Stage II RCC
extends into perirenal spance but still confined within
perirenal fascia of Gerota
Q0753:Stage III RCC
extends into perirenal space with thrombosis to renal vein;
IVC; or lymph nodes
Q0754:which adrenal gland is shaped like pyramind (vs half
moon)
right (left)
Q0755:arterial supply of adrenals
superior suprarenal a (inferior phrenic); middle suprarenal a
(aorta); inferior suprarenal a (renal a)
Q0756:what is opsoclonus?
"dancing eyes"--rapid; irregular; horizontal and vertical
movements; characteristic of neuroblastoma
Q0757:contents of spermatic cord
ductus deferens; testicular artery; artery of ductus deferens;
cremasteric artery; pampiniform plexus of veins;
symp/parasymps; genitofemoral nerve; lymphatics
Q0758:seminal fluid contains what?
fructose and choline
Q0759:prostatic fluid contains
citric acid; acid phosphatase; prostaglandins; fibrinogen; PSA
Q0760:what runs through greater sciatic foramen?
superior gluteal n/a/v; piriformis muscle; inferior gluteal n/a/v;
sciatic n; internal pudendal v/a; pudendal n
Q0761:what runs through lesser sciatic foramen?
internal pudendal a/v; pudendal n
Q0762:what landmarks define pelvic inlet?
sacral promontory (S1); linea terminalis (pubic crest;
iliopectineal line; arcuate line)
Q0763:what structures are in deep perineal space of male?
membranous urethra; urogenital diaphragm (deep transverse
perineal mm; sphincter urethrae mm); brr of internal pudendal
artery (penis artery); brr of pudendal n (dorsal n of penis);
bulbourethral glands
Q0764:what structures are in deep perineal space of female?
urethra; vag; urogenital diaphragm (deep transverse perineal;
sphincter urethrae mm); brr of internal pudendal artery (clit
artery); brr of pudendal n (dorsal clit nerve); NO GLANDS
Q0765:branches of thyrocervical trunk?
(off subclavian); suprascapular artery; transverse cervical
artery; inferior thyroid artery
Q0766:branches off axillary artery?
thoracoacromial; lateral thoracic; anterior humeral circumflex;
posterior humeral circumflex; subscapular
Q0767:branches of brachial artery
deep brachial artery (midshaft fx! with radial nerve); radial
artery; ulnar artery
Q0768:superior ulnar collateral artery anastomoses with ?
posterior ulnar recurrent artery (elbow)
Q0769:inferior ulnar collateral artery anastomoses with ?
anterior ulnar recurrent artery
Q0770:middle collateral artery anastomoses with ?
interosseus recurrent artery
Q0771:radial collateral artery anastomoses with?
radial recurrent artery
Q0772:5 major terminal branches of brahcial plexus
musculocutaneous n (c5-C7); axillary n (C5; C6); radial n (C5-
T1); median n (C5-T1); ulnar n (C5-T1)
Q0773:Klumpke injury involves what rami and damages what
nerves
(lower trunk injury); ventral primary rami of C8; T1; damages
ulnar and median nerves and sympathetic componeny of
spinal nerve T1
Q0774:Erb-Duchenne palsy involves what rami and damages
what nerves?
ventral primary rami of C5; C6; damages musculocutaneous
(biceps; brachialis); suprascapular n (infraspinatus m); axillary
n (teres minor; etc); phrenic n
Q0775:which nerve may be injured by fracture of surgical
neck of humerous or anterior dislocation of shoulder?
axillary n
Q0776:what happens if surgical neck of humerous fractured?
axillary n damage--> abduction of arm TO horizontal impaired
(deltoid); lateral rotaiton of arm weakened (teres minor);
sensory loss of lateral arm
Q0777:which nerve may be injured by badly fitting crutch or
midshaft fx of humerus?
radial nerve (->wrist drop)
Q0778:what happens if midshaft of humerus fractured?
radial n damage--> wrist/finger extensors don't work but
forearm can extend (triceps still ok); sensory loss of posterior
arm/forearm and lateral hand dorsum
Q0779:which nerve may be injured by supracondylar fracture
of humerus?
median nerve (get benediction hand)
Q0780:hand deviates to ulnar side on flexion suggests damage
to which nerve?
median nerve (perhaps supracondylar fx of humerus?)
Q0781:how test for integrity of median nerve?
have pt make OK sign with thumb and index finger
Q0782:how does median nerve injury at wrist manifest
differently than median nerve injury at elbow?
if the damage is at wrist; flexion of thumb is maintained b/c
flexor pollicis longus muscle is spared (if at elbow; thumb
abduction; opposition AND flexion are lost)
Q0783:fracture of medial epicondyle of humerus might
damage what nerve?
ulnar nerve (mild claw hand)
Q0784:slashing of wrist on ulnar side might injure what
nerve?
ulnar nerve (severe claw hand)
Q0785:tendons of what muscles form rotator cuff?
subscapular (subscapular n); infraspinous (suprascapular n);
teres minor (axillary); supraspinous (suprascapular)
Q0786:acute rotator cuff tear usually involves which muscle?
supraspinatous (tendon)
Q0787:what is Nursemaid elbow?
severe distal traction of radius (e.g. caused by parent yanking
child's arm); can cause subluxation of head of radius from
annular ligament
Q0788:what important structures in cubital fossa?
median nerve; brachial artery; biceps brachii tendon; median
cubital v; radial nerve (deep to brachioradialis muscle)
Q0789:adduction at MP joint accompished by?
palmar interosseous muscles
Q0790:aBduction at MP joint accomplished by?
dorsal interosseus muscles)
Q0791:flexion at MP jnt accomplished by?
felxor digitorum superficialis; flexor digitorum profundus;
lumbricals
Q0792:flexion at DIP jnt accomplished primarily by?
flexor digitorum profundus muscle
Q0793:what structures pass through carpal tunnel
flexor digitorum superficialis tendon; flexor digitorum
profundus tendon; flexor pollicis longus tendon; median nerve
(NO ARTERIES)
Q0794:what structures can be damaged in suicide cut on radial
side of wrist?
radial artery; median nerve; flexor carpi radialis tendon;
palmaris longus tendon
Q0795:deep laceration on ulnar side of wrist may cut who?
ulnar artery; ulnar n; flexor carpi ulnaris tendon
Q0796:most commonly fractured carpal bone?
scaphoid
Q0797:what tendons form anatomic snuffbox?
extensor pollicis longus; extensor pollicis brevis; abductor
pollicis longus
Q0798:what is a Colles fracture?
posterior displacement of distal portion of radius. usually
happens when person falls on outstretched hand with wrist
extended; often also fracture of ulnar styloid process
Q0799:significance of artery of ligamentum teres?
artery to the head of femur that is important to children b/c it
supplies the head of the femur proximal to epiphyseal growth
plate. after the growth plate closes; this artery becomes
insignificant
Q0800:in adults; what artery provides the main blood supply
to the head and neck of femur?
medial circumflex artery (branch of profunda femoris)
Q0801:branches of profunda femoris artery (3)
four perforating arteries; medial circumflex artery; lateral
circumflex artery
Q0802:branches of popliteal artery (continuation of femoral a
at adductor hiatus)
genicular arteries; anterior tibial artery (with deep peroneal
nerve); posterior tibial artery
Q0803:what is the cruciate anastomosis?
collateral circulation around hip jnt involving: inferior gluteal
a; medial/lateral femoral circumflex arteries; first perforating
branch of profundus femoris
Q0804:great saphenous vein travels with what nerve around
which malleolus?
travesl with saphenous nerve anterior to MEDIAL malleolus
Q0805:lesser saphenous vein travels with what nerve around
what malleolus?
travels with sural nerve and passes POSTERIOR to
LATERAL malleolus
Q0806:which 2 nerves make up sciatic nerve?
tibial nerve and common peroneal nerve
Q0807:sensory loss of femoral n injury?
anterior thigh and medial leg
Q0808:positive Trendelenburg sign seen with injury to what
nerve?
contralateral superior gluteal nerve
Q0809:fracture of neck of fibular might damage which nerve?
common peroneal n
Q0810:trauma to popliteal fossa might damage what nerve?
tibial nerve
Q0811:clinical features of common peroneal nerve injury
foot drop; inability to stand on heels; walk with foot slap
Q0812:clnical features of tibial nerve injury
inability to stand on toes; dorsiflexion/eversion of foot (b/c
inversion/plantar flexion lost)
Q0813:clinical features of inferior gluteal nerve injury
leaning backward at heel strike
Q0814:sensory loss in tibial nerve injury
sole of foot
Q0815:largest ligament of hip?
iliofemoral ligament (reinforces hip joint anteriorly)
Q0816:which meniscus is C-shaped and more easily torn?
medial
Q0817:what ligaments make up deltoid ligament?
(medial malleolus) anterior tibiotalar; posterior tibiotalar;
tibionavicular; tibiocalcaneal
Q0818:inversion/eversion of foot occurs at what joint?
subtalar
Q0819:what ankle ligament most commonly stretched/torn?
anterior talofibular ligament (of lateral ligament)
Q0820:what is a Jones fracture
avulsion of tuberosity of 5th metatarsal; commonly seen in
inversion injury to ankle
Q0821:specific injuries in Pott fracture (eversion injury)
avulsion of medial malleolus (deltoid ligament v strong);
fracture of fibula
Q0822:contents of anterior triangle (neck) (6)
common carotid artery; IJV; CNX; ansa cervicalis;
sympathetic trunk; CN XII
Q0823:contents of posterior cervical triangle (6)
subclavian artery; EJV; cervical plexus; brachial plexus trunks;
phrenic nerve; CN XI
Q0824:5 major cartilages of larynx
cricoid; thryoid; epiglottis; 2 arytenoids
Q0825:all intrinsic muscles of larynx innervated by WHAT
and what is the exception?
all but cricothyroid muscle (ext branch of superior laryngeal
nerve of CNX) are innervated by inferior laryngeal nerve of
CN X
Q0826:what is the only muscle that ABducts the vocal folds?
posterior cricoarytenoid muscle
Q0827:arterial supply of thyroid gland
superior thyroid artery (external carotid); inferior thyroid
artery (subclavian's thyrocervical trunk); thyroid ima artery
(from aortic arch in 10% of pop.)
Q0828:what is the ligament of Berry?
superior suspensory ligament of thyroid gland
Q0829:surgery on parotid gland may damage what nerve?
auriculotemporal nerve of CN V--> loss of sensation in
auriculotemporal area
Q0830:what is Frey syndrome
sweating whenever a person eats; due to aberrant regeneration
of auriculotemporal nerve of CN V
Q0831:what nerve passes through foramen rotundum?
CN V2
Q0832:what nerve passes through foramen ovale
CN V3 and lesser petrosal nerve
Q0833:what nerve passes through foramen lacerum
trick quesiton. this one is empty
Q0834:which nerves pass through jugular foramen?
CN IX; X; XI
Q0835:what structure is derived from the precordial plate?
the mouth
Q0836:what is the only organ supplied by the foregut artery
that is of mesodermal origin?
spleen
Q0837:in which direction and how far does the gut rotate?
counterclockwise 270 degrees
Q0838:what structure connects the primative gut to the yolk
sac?
yolks stalk;(vitelline duct)
Q0839:artery of the embryonic foregut?
celiac artery
Q0840:when does the gut herniate out of the embryo?;when
does it return?
6 weeks;10 weeks
Q0841:what can occur if the gut does not return to the
embryo?;(2)
- Omphalocele;- Gastroschists
Q0842:around what structure does the midgut rotate?
superior mesenteric artery
Q0843:3 things that cause the gonad to be a testis?
- testis-determining factor from the Y chromosome;- MIF
from sertoli cells;- Testosterone from leydig cells
Q0844:which neuropore opens first and closes last?
caudal
Q0845:when do the septum primum and septum secundum of
the heart fuse?
after birth
Q0846:what is the cerebral cortex a derivative of?
telencephalon
Q0847:what in embryo;ligamentum teres
umbilical vein
Q0848:what in embryo;ligamentum venosum
ductus venosus
Q0849:what two structures are derived from the fourth aortic
arch?
- arch of the aorta;- right subclavian
Q0850:what structure is derived from the first aortic arch?
maxillary artery
Q0851:what are the urinary bladder and the urethra derived
from?
urogenital sinus
Q0852:what are the pulmonary trunk and the ascending aorta
derived from?
truncus arteriosum
Q0853:what are the common and internal carotids derived
from?
third aortic arch
Q0854:what are the two derivatives of the sixth aortic arch?
- pulmonary arteries;- ductus arteriosus
Q0855:what CN is associated with pharyngeal arch;- first;-
second;- third;- fourth;- fifth;- sixth
- CN-5;- CN-7;- CN-9;- CN-10;- none-it degenerates;- CN-10
Q0856:what is the meckles diverticulum a derivative of?
vitelline duct
Q0857:what is the thymus a derivative of?
diencephalon
Q0858:filament associated w/ what cell;- Desmin;- Vimentin
- Desmin: Muscle;- Vimentin: Mesenchymal
Q0859:what are the 2 amino acids that cross-link elastin?
- Desmosine;- Isodesmosine
Q0860:substance in eosinophils that is toxic to parasitic
worms?
Major basic protein
Q0861:what muscle type has calmodulin?
smooth
Q0862:what is the area of mitotic activity in the GI?
crypts of Lieberkulhun
Q0863:produces enamel of the teeth
Ameloblast
Q0864:zone of the liver to be affected first by hypoxia
centeral region;(around central vein)
Q0865:where do you first see clara cells in respiratory
system?
terminal bronchioles
Q0866:what is the chromosome number;- G1 phase;- S
phase;- G2 phase;- mitosis;- primary spermatocyte
- 46 (2n);- 46 (4n);- 46 (4n);- 46 (4n) to 46 (2n);- 46 (4n)
Q0867:when is meiosis arrested first in the female eggs?;At
what stage?;When do they begin tfrough the cycle again?
in Utero;at: Meiosis I; Prophase I;begin again: at Puberty
Q0868:when is miosis arrested the second time in the female
egg?
at ovulation in Metaphase II
Q0869:part of the placenta derived from the mother?
decidua basalis
Q0870:only nerve to come off the dorsal surface of the brain
stem?
CN-4
Q0871:type of fiber in the dorsal root?
sensory
Q0872:what tract do the pain and temp fibers run?
spinothalamic
Q0873:what tract do the touch; pressure; vibration senses
run?
Dorsal column-medial lemniscus
Q0874:thalamic relay nucleus for the visual system?
Lateral Geniculate Nucleus;(LGN)
Q0875:muscle of the eye responsible for accomodation
ciliary
Q0876:if there is macular sparing w/ visual deficit; where is
the lesion?
occipital lobe
Q0877:motor relay of the thalamus
Ventolateral Nucleus;(VL)
Q0878:what is only cell type to leave the cerebellum?
purkinje fibers
Q0879:if patient has right-sided cerebellar lesion; which way
would he fall if he closed his eyes?
to the right
Q0880:if patient presents w/ left nystagmus; where is the
lesion?
on right;(nystagmus is named for the fast component;fast
component is to unaffected side)
Q0881:what fluid of inner ear has the same electrolyte content
as the ECF?
perilymph
Q0882:only cell in the cerebellum to have an excitatory NT?
granule cell
Q0883:what does nystagmus look like if cold water is placed
in the right ear?
slow drift to right; fast to left;COWS= Cold Opposite Warm
Same;(named for fast component)
Q0884:type of memory loss with hippocampal lesion?
long-term memory
Q0885:name the tract of the dorsal columns that are for lower
extremities
gracilis;(graceful dancing)
Q0886:function of the superior colliculi?
Site relay to thalamus;(Superior = Site)
Q0887:if the corticobulbar tract for CN-7 and CN-12 were cut
on the right;where would the deficit be?
no deficit;corticobubar tract receives bilateral input from these
CN
Q0888:as the head moves; what casues the eyes to move in
the opposite direction?
vestibular system
Q0889:part of ear sensitive to angular acceleration and
deceleration?
semicircular canals
Q0890:what artery on the homunculus supplies blood to the
trunk and lower extremities?
anterior cerebral artery
Q0891:eye muscle under;- sympathetic control;- parasymp
control
- symp: Dilator;(symp dilates eye);- para: Sphinctor;(para
constricts like a sphinctor)
Q0892:where is the lesion in Hemiballism?
contralateral subthalamic nuclei
Q0893:what area of the brain is known for language
comprehension?
Wernicke's area
Q0894:what part of the cerebellum is affected by alcohol;
causing the "drink walk" effect?
vermis
Q0895:spinal cord injury resulting in flaccid paralysis &
muscle atrophy?
Polio;(bilateral anterior horn lesions)
Q0896:spinal cord lesion resulting in LMN & UMN
problems?
ALS
Q0897:spinal cord lesion resulting in bilateral loss of pain and
temp at the level of the lesion?
syringomyelia
Q0898:most common circle of willis aneurysm; causing CN-3
palsy
Posterior communicating artery
Q0899:at what level (anatomical landmark) is a lumbar
puncture and a nerve block performed?
Lumbar: Iliac crest;Nerve block: Ischial spine
Q0900:dermatome of the penis for sensation and erection
S2; 3; 4 keeps the penis off the floor
Q0901:spinal cord lesion w/ bilateral dorsal column loss
below level of lesion?
Neuro-Syphilis;(Tabes dorsalis)
Q0902:part of the vertebral disk is from notochord?
nucleus pulposus
Q0903:to what level does the spinal cord extend?
L1 - L2
Q0904:muscle that initiates abduction of the arm?
supraspinalis
Q0905:nerve compressed w/ carpal tunnel syndrome
median
Q0906:"foot drop" is caused by what nerve?
common peroneal
Q0907:nerve damaged if pt cant adduct thigh?
obturator
Q0908:what does the right recurrent laryngeal nerve loop
before ascending into the larynx?
right subclavian
Q0909:at what spinal level does the esophagus originate?
C-6
Q0910:where does the right gonadal vein drain?
inferior vena cava
Q0911:where does the left gonadal vein drain?
left renal vein
Q0912:what is the portal triad?
- common bile duct;- hepatic artery;- portal vein
Q0913:what are the only splanchnics in the body that carry
preganglionic parasympathetic nerves?
Pelvic splanchnics;(P=Pelvic=Pregang=Para)
Q0914:what muscles in the hand are innervated by the median
nerve?;(4);What are the other muscles innervated by?
LOAF;- Lumbricales;- Opponens pollicis;- Abductor
pollicis;- Flexor pollicis brevis;All other muscles: Ulnar nerve
Q0915:what muscles in the hand adduct & abducts the
fingers?
PAD & DAB;- Palmar interosseus: ADducts;- Dorsal
interosseus: ABducts
Q0916:4 muscles of the rotator cuff
SITS;Subscapularis;Infraspinatus;Teres minor;Supraspinatus
Q0917:"unhappy triad of the knee"
- medial collateral ligament;- anterior crucite;- medial meniscus
Q0918:lymph drainage of pelvic organs (not gonads) to what
nodes?
internal iliac nodes
Q0919:pulls arytenoids closer to thyroid-> relaxing vocal
ligaments -> decreasing pitch
thyroarytenoids
Q0920:tenses vocal ligaments -> increasing distance between
cartilages -> increasing pitch
cricothyroids
Q0921:adducts vocal ligaments -> closes airway during
swallowing AND allows phonation
lateral cricoarytenoids
Q0922:abducts vocal ligaments
posterior cricoarytenoids
Q0923:stenson's duct
parotid duct (opp upper second molar)
Q0924:aortic arches
1st- maxillary;2nd- stapes;3rd- common and internal
carotid;4th- aortic arch and ;rt. subclavian;5th-
degenerates;6th- rt and lt pulmonary arteries;(MS. CARD and
pulmonary aa)
Q0925:adduct thigh and flex hip
obturator n.
Q0926:plantar flex foot;flex toes;invert foot
tibial n;(tIbial n. -> Inverts)
Q0927:dorsiflex foot;extend toes;evert foot
deep peroneal n;(pEroneal n. -> Everts)
Q0928:flex hip and extend knee
femoral n.
Q0929:extend hip and flex knee
tibial n.
Q0930:plantar flex foot and evert
superficial peroneal n.
Q0931:branches of posterior cord of brachial plexus
STARS;upp
Subscapularis;Thoracodorsal;Axillary;Radial;lower
Subscapularis
Q0932:rt. recurrent laryngeal n. passes under;
rt brachiocephalic a.
Q0933:lt recurrent laryngeal n. passes under;
arch of the aorta
Q0934:inferior mesenteric artery drains into;
splenic vein
Q0935:acetabulum bones
ischium; ilium; pubis
Q0936:epidural hematoma
middle meningeal
Q0937:elevates and abducts eyeball
inferior oblique
Q0938:depresses and abducts eyeball
superior oblique
Q0939:uvula deviation in left vagus nerve lesion
right
Q0940:subdural hematoma vessels
bridging veins
Q0941:intertubercular groove of the humerus mm insertion
latissimus dorsi;teres major;pec major
Q0942:taste to anterior two thirds of tongue
chorda tympani of VII
Q0943:blink reflex
V1 and VII
Q0944:most superior orbit mm
levator palpebrae superioris
Q0945:what part of pericardium adheres to tunica adventita
of great vessels
fibrous
Q0946:two veins that form the portal vein
superior mesenteric and splenic
Q0947:light reflex
II and III (parasympathetic)
Q0948:lumbar puncture
skin;superficial fascia;deep
fascia;supraspinatous;interspinatous;interlaminar
space;epidural space;dural mater;arachnoid
mater;subarachnoid space;(Sally Sue Didn't Scream In
between Intern's Expected Deviations And Screw-ups)
Q0949:sluggish pupil response to light and normal
accomondation
ciliary ganglion -> tonic pupil
Q0950:rotator cuff
SITS;supraspinatus;infraspinatus;teres minor;subscapularis
Q0951:white rami communicantes
preganglionic sympathetic axons
Q0952:suprascapular nerve innervates
supraspinatus;infraspinatus
Q0953:upper subscapularis
subscapularis
Q0954:thoracodorsal nerve
latissimus dorsi
Q0955:long thoracic nerve
serratus anterior
Q0956:flex wrist and digits; pronate wrist; and LOAF mm
median nerve
Q0957:flex shoulder; flex elbow; supinate elbow
musculocutaneous nerve
Q0958:innervates flexor carpi ulnaris; flexor digiti profundus
(pinky and ring fingers); and intrinsic mm of hand
ulnar nerve
Q0959:gag reflex
IX and X
Q0960:vein that drains lower third of thoracic wall
hemiazygous
Q0961:where does the axillary artery become the brachial
artery
when it crosses the teres major
Q0962:tongue deviates in a left CN XII lesion
left
Q0963:vertebral level of common carotid bifurcation
C4
Q0964:who gets femoral hernias
Females
Q0965:where is the profundus femoris artery
anterior compartment even though it supplies the posterior
compartment
Q0966:relationship of the cupula to the subclavian artery and
vein
posterior
Q0967:vertebral level that aortic arch begins and ends
T4
Q0968:which artery travels w/ great cardiac vein
LADA
Q0969:middle cardiac vein
posterior interventricular artery
Q0970:small cardiac vein
right coronary artery
Q0971:what passes at T8?;T10?;T12
T8- one: IVC;T10- two: esophagus and esophageal nerve
plexus;T12- three: aorta; thoracic duct; azygous vein
Q0972:carotid sinus senses;
pressure;(*Sinus Pressure*)
Q0973:taste to the posterior third of the tongue
CN IX
Q0974:sympathetic control of which eye mm
dilator pupillae
Q0975:three branches of lateral cord of brachial plexus
lateral pectoral;lateral head of median;musculocutaneous
Q0976:how do veins in the face differ
have no valves
Q0977:shoulder bony articulations
clavicle;acromion;glenoid fossa of scapula;humerus
Q0978:elbow bony articulations
humerus w/ ulna (major) and radius (minor)
Q0979:wrist bony articulation
radius w/ scaphoid and lunate;ulna w/ triquetrum and pisiform
Q0980:only laryngeal mm innervated by the external laryngeal
nerve
cricothyroid muscle (increases pitch);all the rest are
innervated by recurrent laryngeal nerve
Q0981:how many bronchopulmonary segments are there
10 on the right;8 on the left
Q0982:seven structures found in more than one mediastinum
esophagus;SVC;vagus nerve;azygous vein;thoracic
duct;thymus;phrenic nerve
Q0983:what suspends the duodenal-jejunal flexure from the
posterior abdominal wall
ligament of treitz
Q0984:only tongue mm innervated by X
palatoglossus;all others are innervated by XII
Q0985:five branches of the superior mesenteric artery
inferior pancreaticoduodenal;middle colic;right
colic;ileocolic;10-15 intestinal aa.
Q0986:what innervates the detrusor mm
S2; S3; S4
Q0987:what connects the third and fourth ventricles
cerebral aqueduct
Q0988:what nerve and artery could be affected in a humeral
neck fracture
axillary nerve and posterior humeral artery
Q0989:what hernia passes through the deep lateral ring of
inguial canal
Indirect passes In the Inguinal canal;(direct goes directly
through hasselbachs triangle)
Q0990:what forms the external jugular vein
posterior auricular vn;posterior retromandibular vn
Q0991:three branches of celiac trunk
left gastric;splenic;hepatic
Q0992:where does the eustachian tube enter the pharynx
nasopharynx
Q0993:only mm of soft palate innervated by V3
tensor veli palantini;(all others innervated by X)
Q0994:what artery turns into the dorsalis pedis when it
crosses the extensor retinaculum
anterior tibial
Q0995:what connects the lateral ventricles to the third
ventricle
foramen of monro
Q0996:sensation to anterior two thirds of tongue
lingual branch of V3
Q0997:what type of pleura is adherent to the surface of an
organ
visceral
Q0998:what artery supplies the left ventricle;left atrium; and
interventricular septum
left coronary artery
Q0999:where are the tonsillar tissues
Waldeyer's ring
Q1000:what three characteristics differentiate the large bowel
teniae coli;haustra;epiploic appendages
Q1001:what area of the eye has no photoreceptors
optic disk (blind spot)
Q1002:at the level of rib 6; the internal tghoracic artery
divides into what two arteries
musculophrenic and superior epigastric
Q1003:what is the name of inflammation of prepatellar bursa
housemaid's knee
Q1004:what nerve roots make up the cervical plexus
C1-C4
Q1005:only organ supplied by preganglionic sympathetic
fibers
adrenal medulla
Q1006:left subclavian artery is a branch of;
aortic arch
Q1007:four mm of mastication
masseter;temporalis;medial pterygoid;lateral pterygoid
Q1008:with what thoracic vertebra does rib 7 articulate
T7 and T8;***each rib articulates w/ its corresponding
numerical vetrabral body and the vertebral body BELOW it
Q1009:the three branhces of the inferior mesenteric artery
left colic;sigmoidal;superior rectal
Q1010:only valve in the heart w/ two cusps
mitral valve
Q1011:five clinical signs of portal HTN
caput medussae;esophageal varices;internal
hemerrhoids;retroperitoneal varices;splenomegaly
Q1012:what three mm constitute the erector spinae
Iliocostalis;Longissimus;Spinalis;(I Love Science)
Q1013:what nerve is compromised in carpal tunnel
median nerve
Q1014:what vascular injury may result from supracondylar
fracture of the femur
popliteal artery
Q1015:what nerve and artery could be affected in a midshaft
humeral fracture
radial nerve and profunda brachii artery
Q1016:name the 10 retroperitoneal organs
Duodenum;ascending Colon;Ureters;Pancreas;Suprarenal
gland;Descending colon;Aorta;Kidneys;Rectum;IVC;(D CUPS
DAKRI)
Q1017:what foramen is transversed to get into the lesser
peritoneal sac
foramen of Winslow
Q1018:what goes through the foramen magnum
spinal cord;CN XI;vertebral arteries
Q1019:foramen spinosum
middle meningeal artery
Q1020:foramen rotundum
V2
Q1021:foramen ovale
V3 and lesser petrosal nerve
Q1022:jugular foramen
IX; X; XI; sigmoid sinus
Q1023:carotid canal
internal carotid and ;sympathetic nerve plexus
Q1024:stylomastoid foramen
CN VII
Q1025:hypoglossal canal
CN XII
Q1026:internal auditory meatus
CN VII and VIII
Q1027:optic canal
II and ophtalmic artery
Q1028:cribiform plate
CN I
Q1029:superior orbital fissure
CN III; IV; VI and ophthalmic vein
Q1030:what vessel is atop the scalene anterior
subclavian vein
Q1031:what part of the corneal reflex is lost in a CN VII
deficit
motor
Q1032:jaw deviation in rt CN V lesion
right
Q1033:what two artries join to form the superifcial and deep
palmar arches of the hand
ULNAR and radial
Q1034:what two ligaments of the uterus are remnants of the
gubernaculum
round and ovarian ligaments
Q1035:what forms the tibial nerve
L4 to S3;(L2-L4 does the thigh;L4-S3 does the leg)
Q1036:what forms the common peroneal nerve
L4 to S3;(L2-L4 does the thigh;L4-S3 does the leg)
Q1037:what forms of femoral nerve
L2-L4;(L2-L4 does the thigh;L4-S3 does the leg)
Q1038:what forms the obturator nerve
L2-L4;(L2-L4 does the thigh;L4-S3 does the leg)
Q1039:what three structures are in contact w/ the left colic
flexure?;right colic flexure?
spleen; stomach; left kidney;liver; duodenum; right kidney
Q1040:what three muscles constitute the pes anserinus
sartorius;gracilis;semitendinous
Q1041:only pharyngeal muscle NOT innervated by X
stylopharyngeus is innervated by IX
Q1042:fracture of the fibular neck resulting in foot drop
damaged what nerve
common peroneal
Q1043:does the VAN run on the inferior or superior side of a
rib
inferior side
Q1044:what muscle laterally rotates the femur to unlock the
knee
popliteus
Q1045:what chamber of the eye lies between the iris and the
lens
posterior chamber
Q1046:what artery supplies the right atrium;right
ventricle;SA node;AV node
right coronary artery
Q1047:what four branches of the brachial plexus arise prior to
the first rib
dorsal scapular;suprascapular;long thoracic;nerve to
subclavius
Q1048:the xiphoid process marks which vertebral level
T9
Q1049:loss of eversion; inversion; dorsiflexion; and plantar
flexion
common peroneal nerve
Q1050:loss of flexion of knees and toes; plantar flexion; and
weakened inversion
tibial nerve
Q1051:loss of knee extension and weakened hip flexion
femoral nerve
Q1052:loss of abduction of hip resulting in Trendelenburg gait
superior gluteal nerve
Q1053:loss of flexion of the knee and all function below the
knee; weakened extension of the thigh
sciatic nerve
Q1054:loss of adduction of thigh
obturator nerve
Q1055:what nerve lesion presents w/ ape or simian hand as
its sign
median nerve lesion
Q1056:first branch of the abdominal aortic artery
inferior phrenic artery
Q1057:what vessel does right gonadal vein drein into
right gonal vein drains into IVC;left drains into left renal vein!!
Q1058:what two muscles do you test to see of CN XI is
intact
trapezius and sternocleidomastoid
Q1059:what two CNs are responsible for the carotid body
and sinus relfexes
IX and X
Q1060:at what vertebral vels does the trachea bifurctae
T4 (anteriorly it is the angle of Louis)
Q1061:what is the function of the arachnoid granulations
resorb CSF into blood
Q1062:winged scapula
long thoracic nerve
Q1063:what portion of the intervertebral disck is a remnant of
the notochord
nucleus pulposus
Q1064:what component of the pelvic diaphragm forms the
rectal sling (muscle of continence?)
puborectalis
Q1065:five branches of Median nerve of brachial plexus
Median;Medial antebrachial;Medial pectoral;Medial brachial
cutaneous;Ulnar
Q1066:what bone houses the ulnar groove
humerus (between the medial epicondyle and the trochlea)
Q1067:sensory innervation to the
nasopharynx?;oropharynx?;laryngopharynx?
V3 and glossopharyngeal;glossopharyngeal;vagus
Q1068:what protective covering adheres to the spinal cord
and CNS tissue
pia mater
Q1069:what is the name of the urinary bladder where the
uterers enter and the urethra exits
trigone
Q1070:what is the term when the brachial artery is
compressed; resulting in ischemic contracture of the hand
Volkmann's contracture
Q1071:what attaches the cusps of the valves to the papillary
mm in the heart
chordae tendineae
Q1072:what is the lymphatic drainage of the pelvic organs
internal iliac nodes
Q1073:what bursa is inflamed in clergyman's knee
infrapatellar bursa
Q1074:chief flexor of the hip
psoas major
Q1075:what muscles in the hand adduct the fingers
Palmar interosseus ADducts;(PAD);(the Dorsal interosseus
ABducts- DAB)
Q1076:what type of cerebral bleed is due to a rupture of the
berry aneurysm in the circle of willis
subarachnoid hematoma
Q1077:what are the five termian branches of the facial nerve
temporal;zygomatic;buccal;mandibular;cervical;(two zebras
bit my clavicle)
Q1078:cough reflex
superior larnygeal nerve of X;recurrent laryngeal nerve of X
Q1079:sensory innervation above the vocal cords?;below?
internal laryngeal nerve ;recurrent laryngeal nerve
Q1080:C-shaped shock absorber; aids in attachment of tibia
to femur via medial collateral ligament
medial meniscus
Q1081:prevents posterior displacement and has medial to
lateral attachment on tibia
PCL
Q1082:prevents adduction of knee joint
lateral collateral ligament
Q1083:prevents abduction
medial collateral ligament
Q1084:dye passes from one cell to the next
gap junction
Q1085:derivation of adrenal medulla
neural crest origin (S100);neuroblasts develop into ganglia
Q1086:tibial nerve function
plantar flexion of toes and inversion
Q1087:composition of aortic valve and pulmonic valve
lined by endothelium and have abundant fibroelastic tissue
plus a dense collagen core; avascular;MV and TV have a loose
connective tissue core which is increased in MV/TV prolapse
(myxomatous degeneration)
Q1088:break humerus; wrist drop
radial nerve injury
Q1089:post radical mastectomy- winged scapula
long thoracic nerve -> paralysis of serratus anterior
Q1090:nosebleed and rhinorrhea
fracture of cribiform plate in ethmoid sinus
Q1091:medial longitudinal fasculus demyelination in MS
bilateral internuclear ophthalmoplagia
Q1092:parathyroid derivation
third and fourth pharyngeal puch
Q1093:aortic arch derivates
M;S;C;A/R;D;pulmonary arteries
Q1094:what runs along the radial artery
median nerve
Q1095:artery affected in femoral neck fracture
medial femoral circumflex artery
Q1096:EM of egg- where does sperm penetrate?
zona pellucida
Q1097:where is metaphase II completed
uterus
Q1098:bone;tendon;skin;greatest tendon strength
collagen type 1
Q1099:initial wound repair;replaced by type I
type III
Q1100:basement membrane
type IV
Q1101:epiphyseal plate
type X (picture of bone; point where collagen type X is)
Q1102:circle of willis
draw him!
Q1103:CT liver
know where the hepatic vein drains into the
IVC;http://www.med.wayne.edu/diagradiology/Anatomy_Mo
dules/Abdomen.html
Q1104:nerve injured in midshalft humerus frx
radial nerve
Q1105:layers of gastric mucosa
MUCOSA;epithelium;lamina propria;muscularis
mucosa;SUBMUCOSA;MUSCULARIS;SEROSA
Q1106:what myelinates in the CNS?;in the periphery?
oligodendrocytes;schwann cells
Q1107:stage of eggs post-partum
meiosis I arrested in prophase
Q1108:eye closed can't open; eye deviated down and out
CN III palsy
Q1109:eye down and in
CN IV nerve palsy
Q1110:child w/ popsicle stick in his mouth falls down causing
the popsicle stick to hit the back of his throat and develops
ptosis and meiosis of right eye
injury to the cervial sympathetic ganglion
Q1111:vertical diplopia
CN IV palsy
Q1112:patient with headache and physical findings of
mydriasis in right eye in association w/ lid lag; and deviation
of eye down and out
aneurysm compressing CN III
Q1113:patient with a recent history of bacterial meningitis
has horizontal diplopia in the left eye which is worse on gaze
to the left
CN VI palsy
Q1114:patient with bilateral lateral rectus muscle weakness
increase intracranial pressure;(papilledema usual present)
Q1115:pralysis of upward gaze in an infant
hydrocephalus secondary to senosis of aqueduct of sylvius
(=parinaud's syndrome)
Q1116:multiple ocular motor nerve disorders
diabetes mellitus
Q1117:weakness of quadriceps and an absent knee jerk reflex
herniated L3-L4 disk
Q1118:pain in hip and lateral quadriceps; numbness of
anterolateral leg and webbed space betweeng reat toe;
weakness of dorsiflexion of foot; normal reflexes
hernated L5-S1 disk
Q1119:young child falls on his outstretched arm and has pain
in the middle and lateral portion of his clavicle; upper
extremity remains in abduction; extension; and internal
rotation
C5-C6 = Erb-Duchenne syndrome -> superior brachial plexus
injury due to clavicular fracture;*most common frx in infants
Q1120:patient has pralysis of the oculomotor nerve after a
head injury
uncal herniation with compression of CN III
Q1121:numbness of thenar aspect of hand
median nerve (carpal tunnel)
Q1122:wrist bone with greates incidence of aseptic necrosis
navicular bone (scaphoid)
Q1123:supracondylar fracture
injury to brachial artery andmedian nerve; danger of ischemic
contractures in forearm mucle (Volkmann's ischemic
contracture)
Q1124:know the fetal circulation- which vessels have the
highest oxygen content
ductus venosus and umbilical vein
Q1125:EM of alveolus and macrophage
type II pneumocyte with lamellar bodies making surfactant!
Q1126:EM of small bowel
microvilli
Q1127:histosection of seminiferous tubule- what cell makes
sex hormone binding globulin and inhibin
sertoli cell;http://www.lab.anhb.uwa.edu.au/mb140/
Q1128:know the bands in skeletal mm
http://www.lab.anhb.uwa.edu.au/mb140/
Q1129:voice hoarseness post thyroid surgery
injury to laryngeal nerve
Q1130:MRI of orbit- can you find the superior oblique
muscle
no :(
Q1131:MRI of abdomen- can you find splenic artery above
the pancreas
no :(
Q1132:fluid in costophrenic sulcus on CXR
CHF
Q1133:x-ray showing elargement of the posterior heart
enlarged left atrium in a petient w/ mitral stensois
Q1134:mri of carotids w/ occlusion of the anterior cerebral
artery would effect which leg?
contralateral
Q1135:schwannoma in jugular foramen would effect which
three CN
IX;X;XI;loss of gag reflex;laryngeal
paralysis;trapezius/sternocleidomastoid;loss of taste in
posterior third of tongue
Q1136:lesion of oculomotor nerve and UMN signs with a
midline; midbrain lesion
weber syndrome
Q1137:horner's syndrome- diagram of vertebra and
sympathetic trunk- pick out the damaged ganglia
cervical sympathetic ganglion
Q1138:groos of brainstem anterior view- find area of
oculomotor nerve
-
Q1139:frontal lobe lesion affects
personality
Q1140:bitemproal hemianopsia
leison at optic chiasm;commonly a caniopharyngeiona
(derived from rathke's ouch)
Q1141:inferior quadrantanopia
defect in superior fibers in parietal lobe
Q1142:C2 transection of fascuculs gracilis
vibration and fine touch of LE only
Q1143:CN III and UMN signs on opposite side
midline brain lesion
Q1144:loss of pain and tem and UMN signs on opposite side
mid pons lesion
Q1145:Horner's syndrome with localization of lesion on
diagram
lateral medullary syndrome w/ assoc cranial nerve palsy in
medulla and hypothalamus w/ assoc temp regulation problems
Q1146:Parkinson's drug of abuse
MPTP
Q1147:blood preduction prior to birth
liver;bone marrow
Q1148:bochdalek hernia in posterolateral diaphragm on left
early in life; visceral contents extend into chest cavity ->
respiratory distress
Q1149:parasternal diaphragmatic hernia
present later in life;go through foramen of morgangni beneath
the sternum
Q1150:artery assoc w/ foregut;midgut;hindgut
celiac;superior mesenteric;inferior mesenteric
Q1151:damaged hearing in rock and roll band player
injured cochlea
Q1152:hypospadias
faulty closure of urethral folds
Q1153:epispadia;assoc w/
faulty closure of genital tubercle;assoc w/ bladder exstrophy
Q1154:feces draining from umbilicus;urine draining from
umbilicus?
persistant umbilical (vitelline) sinus;persistant urachal sinus
Q1155:falciform ligament--connects? contains?
liver to anterior ab wall; ligamentum teres (remnant of L
umbilical vein;R vein obliterated during embryonic period)
Q1156:hepatuduodenal ligament contains?
portal triad: hepatic artery; portal vein; common bile duct
Q1157:gastrohepatic ligament contains?
gastric arteries (may be cut during surgery to access lesser sac)
Q1158:gastrocolic ligament contains?
gastroepiploic arteries (part of greater omentum)
Q1159:gastrosplenic ligament contains?
short gastric vessels; L gastroepiploic vessels
Q1160:splenorenal ligament contains?
terminal brances of splenic artery; tributaries of splenic vein;
tail of pancreas
Q1161:Muscles of the abdominal wall
External oblique; internal oblique; transverse; rectus
abdominus; transversalis fascia
Q1162:What structures form the roof of the inguinal canal?
Internal abdominal oblique and transverse abdominus muscles
Q1163:What structures form the anterior wall of the inguinal
canal?
Aponeurosis of the external and internal abodminal oblique
muscles
Q1164:What structures form the floor of the inguinal canal?
Inguinal ligament
Q1165:What structures form the posterior wall of the inguinal
canal?
Transversalis fascia (laterally) and cojoined tendon (medial)
Q1166:What are the contents of the spermatic cord and where
do they derive from?
External spermatic fascia (derives from external oblique
fascia); middle spermatic fascia and cremaster muscle (internal
oblique muscle); internal spermatic fascia (transversalis
fascia); testicular artery; Pampiniform venous plexus; vas
deferens
Q1167:Where and what forms the deep inguinal ring?
Its formed by the transversalis fascia just lateral of the inferior
epigastric vessels
Q1168:Direct inguinal hernia
Protrussion through the posterior wall of inguinal canal medial
to the inferior epigastric vessels and deep inguinal ring
Q1169:Indirect inguinal herna
Protrussion through the deep ingiunal ring and courses
through the inguinal canal
Q1170:What is the cause of a congenial indirect inguinal
hernia?
Persistence of the process vaginalis
Q1171:MCC site of femoral hernia
Medial from femoral vein just below inguinal canal. MC in
women.
Q1172:Bones of the posterior abdominal wall
Ileum (iliac crest; iliac fossa); five lumbar vertebra; 12th rib;
lesser trocanter of femur
Q1173:Muscles of the posterior abdominal wall
Psoas; quadratus lumborum; iliacus
Q1174:Muscle that flexes the hip
Psoas major
Q1175:Lobes of the liver
Right and left lobes divided by the falciform ligament. Right
lobe is subdivided into quadrate; proper and caudate lobes
Q1176:Omental bursa
Cavity behind the stomach; has epiploic foramen
Q1177:Structures of the epiploic foramen
Anteriorly the [prtal vein and hepatoduodenal ligament.
Posteriorly the inferior vena cava.
Q1178:Structures and associations of the portal triad
Portal vein; hepatic artery and common bile duct. Associated
with the hepatoduodenal ligament and the free border of the
lesser omentum
Q1179:How is the common bile duct formed?
By the cystic (gallbladder) duct and the common hepatic duct
Q1180:What are the pancreatic ducts and where do they
open?
Major duct of Wirsung (developes from ventral pancreatic
diverticulum) and minor duct of Santorini (develop from
dorsal pancreatic diverticulum). They open into the 2nd
portion of the duodenum.
Q1181:Irrigation of the pancreas
The head is irrigated by superior and inferior
pancreaticoduodenal arteries. The body and tail from the
splenic arteries.
Q1182:Major branches of the abdominal aorta
Celiac trunk; superior mesenteric; right and left renal arteries.
It bifurcates into the common iliac arteries which further
divide into the internal and external iliac arteries.
Q1183:Branches of the celiac trunk
Left gastric artery (lesser curvature of stomach); splenic
artery; common hepatic artery
Q1184:Branches of the common hepatic artery
Proper hepatic artery and gastroduodenal arteries.
Gastroduodenal artery branches into gastroepiploic artery
(major curvature of stomach) and pancreaticoduodenal artery.
Q1185:Irrigation of the colon
Cecum; ascending colon and proximal 2/3 of the transverse
colon irrigated by superior mesenteric artery. Distal 1/3 of the
transverse colon; descending colon; sigmoid and rectum
irrigated by inferior mesenteric artery
Q1186:Branches of the superior mesenteric artery
Ileocolic; right colic and middle colic arteries.
Q1187:Branches of the inferior mesenteric artery
Left colic; sigmoid and superior rectal arteries
Q1188:What nerve roots form the superior trunk of brachial
plexus?
C5; C6
Q1189:What nerve roots form the middle trunk of the brachial
plexus?
C7
Q1190:What nerve roots form the inferior trunk of the
brachial plexus?
C8; T1
Q1191:How is the axillary nerve formed?
It's the superior branch of the posterior cord
Q1192:Muscles innervated by the axillary nerve
Deltoid and teres minor
Q1193:Muscles innervated by the musculocutaneous nerve
Biceps; brachial; choracobrachial
Q1194:Muscles innervated by the radial nerve
Triceps and extensor muscles on the posterior compartments
of arm and forearm
Q1195:Muscles innervated by the ulnar nerve
Flexor carpi ulnaris; medial part of the flexor digitorus
profundus; lumbricals 3 and 4; palmar interosseous; dorsal
interosseous; adductor pollicis; abductor digiti minimi; flexor
digiti minimi; opponens digiti minimi
Q1196:Muscles innervated by the median nerve
Flexor muscles of the forearm except flexor carpi ulnaris and
medial part of flexor digitorus profundus; lumbricals 1 and 2;
abductor pollicis brevis; opponens pollicis brevis; flexor
pollicis brevis.
Q1197:Actions of the axillary nerve
Abduct shoulder (deltoid); lateral roation of the shoulder
(teres minor)
Q1198:Actions of the musculocutaneous nerve
Flexes elbow (biceps; chorachobrachial); supination (biceps
brachii)
Q1199:Actions of the radial nerve
Extends digits; wrist and elbow. Supination.
Q1200:Actions of the ulnar nerve
Flexes wrist and digits 4 and 5. Abducts and adducts digits 2;
3; 4; 5.
Q1201:Actions of the median nerve
Flexes wrist and digits; pronation; abduction; opposition and
flexion of the thumb
Q1202:Clinical features of injury of the upper trunk of the
brachial plexus
C5; C6 are injured due to adduction traction of the arm and
hyperextension of the neck (fall on shoulder or pull baby by
the neck). Results in Erb-Duchene paralysis damaging axillary;
musculocutaneous; suprascapular and phrenic nerves. Loss of
shoulder and anterior arm muscles. Arm is medially rotated
and adducted; forearm is extended and pronated (waiter's tip)
Q1203:Clinical features of injury of the lower trunk of the
brachial plexus
C8; T1 are injured by a sudden abduction upward pull of the
arm (pull baby by the arm). Results in thoracic outlet
syndrome damaging median and ulnar nerves as well as T1
sympathetics. Loss of muscles of forearm and hand and
Horner's syndrome (ptosis; miosis; anhydrosis)
Q1204:Long thoracic nerve injury
Paralysis of the serratus anterior. No abduction of the arm
past the horizontal position. Can't push (winging of the
scapula)
Q1205:Axillary nerve injury
Injured by fracture at the surgical neck of humerus or anterior
dislocation of the shoulder. Cant abduct the shoulder to the
horizontal plane; cant lateral rotate shoulder (teres minor).
Sensory loss at the shoulder.
Q1206:Radial nerve injury at the axilla
Shoulder dislocation or pressure on the floor of the axilla. No
extension of the elbow; wrist or digits. Weak supination.
Sensory loss of posterior arm; posterior forearm and
posterolateral hand. "Wrist drop"
Q1207:Radial nerve injury at the arm
Midshaft fracture of the humerus. No extension of the wrist
or digits. Sensory loss on the posterior forearm and
posterolateral hand. "Wrist drop"
Q1208:Radial nerve injury at the wrist
Sensory loss on the lateral part of the dorsum of the hand.
Q1209:Median nerve injury at the elbow
Supracondylar fracture of the humerus. Weak flexion of the
wrist with ulnar deviation upon flexion. No flexion of digits 1
and 2; thenar muscles and lumbricals 1 and 2. Sensory loss on
lateral palm and fingers 1; 2 and 3. Flattening of the thenar
eminence (ape hand) and index and middle fingers cant make a
fist.
Q1210:Median nerve injury at the wrist
Slashing of the wrist (suicide attempt). Loss of thenar muscles
and lumbricals 1 and 2. Sensory loss on lateral palm and
fingers 1; 2 and 3. Flattening of the thenar eminence (ape
hand) and index and middle fingers cant make a fist.
Q1211:Ulnar nerve injury at the elbow
Fracture of the medial epicondyle of the humerus. Hand
deviates radially when flexed (injured flexor carpis ulnaris); no
flexion of ring and little fingers (injured flexor digitorus
profundus and lumbricals 3; 4); abduction and adduction of
the fingers is lost (injured palmar and dorsal interosseous); no
adduction of the thumb; no mevement of little finger. Sensory
lost on lateral half of ring finger and little finger. "Claw hand".
Q1212:Ulnar nerve injury at the wrist
Slashing of the wrist (suicide attempt). Loss of abduction and
adduction of the digits; loss of hypothenar muscles and
lumbricals 3; 4. Sensory loss on lateral half of digit 4 and 5.
"Claw hand".
Q1213:Musculocutaneous nerve injury
Loss of elbow flexion and weak suppination. Loss of
sensation on lateral aspect of the forearm.
Q1214:Branches of the brachiocephalic trunk
Left subclavian and left common carotid arteries
Q1215:Branches of the subclavian artery
Internal thoracic artery (continuous with superior epigastric
and inferior epigastric which provide colateral circulation in
postductal coarctation of the aorta); vertebral artery.
Q1216:Branches of the axillary artery
Thoracoacromial; lateral thoracic (mammary gland);
subscapular (collateral to shoulder); posterior and anterior
humeral circumflex arteries.
Q1217:Branches of the brachial artery
Deep brachial artery (together with radial nerve at midshaft of
humerus); ulnar artery and radial artery at the cubital fossa.
Q1218:Branches of the ulnar artery
Common interosseous artery and superficial palmar arch
Q1219:Branches of the radial artery
Deep palmar arch
Q1220:Rotator cuff muscles
Subscapularis; Infraspinatous; Teres minor; Supraspinatous.
"SITS"
Q1221:What are the joints of the elbow and what is the
function of each?
Humeroulnar and humeroradial permit flexion and extension.
The radioulnar permits pronation and supination.
Q1222:Wrist and hand joints
Radiocarpal joint (between the distal radius and
scaphoid/lunate carpal bones); ulnocarpal joint; midcarpal
joint (between proximal and distal carpal bones);
carpometacarpal joints.
Q1223:What are the carpal bones
From lateral to medial. Proximal row: scaphoid; lunate;
triquetrum; pisiform. Distal row: trapezium; trapezoid;
capitate; hamate.
Q1224:What are the structures in the carpal tunnel?
Flexor digitorum superficialis (4 tendons); flexor digitalis
profundus (4 tendons); flexor pollicis longus and median
nerve.
Q1225:Fracture of the scaphoid bone
Results in avascular necrosis of the proximal head of the
scaphoid.
Q1226:Lunate bone dislocation
Results in median nerve compression in the carpal tunnel
Q1227:triceps surae
gastrocnemius and soleus
Q1228:gastrocnemius
plantar flexes the foot and flexes the leg
Q1229:soleus
plantar flexes the foot
Q1230:tibialis anterior
dorsiflexes and inverts the foot
Q1231:tibialis posterior
plantar flexes the foot and inverts
Q1232:peroneus longus
plantar flexes and everts foot
Q1233:peroneus brevis
plantar flexes and everts foot
Q1234:quadriceps femoris
rectus femoris; vastus lateralis; vastus medialis; vastus
intermedius
Q1235:rectus femoris
flexes the thigh and extends the leg
Q1236:vastus lateralis
extends the leg
Q1237:vastus medialis
extends the leg
Q1238:vastus intermedius
extends the leg
Q1239:sartorius
flexes the leg; flexes and laterally rotates the thigh (sitting
cross legged)
Q1240:hamstrings
biceps femoris; semimembranous; semitendinosus
Q1241:biceps femoris
flexes the leg and extends the thigh
Q1242:semimembranosus
flexes the leg and extends the thigh
Q1243:semitendinosus
flexes the leg and extends the thigh
Q1244:iliopsoas
iliacus and psoas major
Q1245:iliacus
flexes the thigh
Q1246:psoas major
flexes the thigh
Q1247:tensor fascia latae
flexes and abducts the thigh
Q1248:gluteus maximus
extends and laterally rotates the thigh
Q1249:gluteus medius
abducts and medially rotates the thigh
Q1250:gluteus minimus
abducts and medially rotates the thigh
Q1251:gracilis
adducts the thigh and flexes the leg
Q1252:adductor magnus
adducts and laterally rotates the thigh (more posterior)
Q1253:adductor longus
adducts and flexes and laterally rotates the thigh (more
anterior)
Q1254:adductor brevis
adducts and laterally rotates and flexes the thigh
Q1255:hand flexors
flex the hand (anterior)
Q1256:hand extensors
extend the hand (posterior)
Q1257:biceps brachii
flexes and supinates the forearm
Q1258:brachialis
flexes the forearm
Q1259:brachioradialis
flexes the forearm
Q1260:triceps brachii
extends the forearm
Q1261:pectoralis major
flexes; adducts; and medially rotates the arm
Q1262:latissiumus dorsi
adducts; extends; and medially rotates the arm
Q1263:deltoid
abducts and flexes the arm
Q1264:supraspinatus
abducts the arm
Q1265:infraspinatus
laterally rotates the arm
Q1266:teres major
extends; adducts; and medially rotates the arm
Q1267:rotator cuff
subscapularis; infraspinatus; teres minor; supraspinatus
(SITS)
Q1268:pectoralis minor
depresses and protracts the scapula
Q1269:trapezius
adducts; elevates; depresses scapula; extends the head; elevate
the clavicle
Q1270:levator scapulae
elevates the scapula
Q1271:rhomboid major
adducts and inferiorly rotates the scapula
Q1272:rhomboid minor
adducts and inferiorly rotates the scapula
Q1273:rectus abdominus
flexes the vertebral column and compresses the abdomen
Q1274:external oblique
rotation of the trunk; compression of the abdomen
Q1275:internal oblique
rotation of the trunk; compression of the abdomen
Q1276:transverse abdominis
compression of the abdomen
Q1277:erector spinae muscles
iliocostalis; longissimus; spinalis
Q1278:function of erector spinae muscles
ind: laterally flex and rotate the vertebral column;group:
muscles extend the vertebral column
Q1279:diaphragm
moves downward during inspiration; increase dimensions of
the thorax
Q1280:external intercostals
pulls ribs upward and together to expand thorax (labored
inspiration)
Q1281:internal intercostals
pulls ribs downward and together to decrease thorax (forced
expiration)
Q1282:epicranius frontalis
raises the eyebrows
Q1283:epicranius occipitalis
draws scalp posteriorly
Q1284:orbicularis oris
puckers mouth as in kissing
Q1285:orbicularis oculi
closes each eye as in blinking
Q1286:zygomaticus
pulls the corners of the mouth as in smiling
Q1287:platysma
depresses the lower lip and mandible
Q1288:temporalis
elevates and retracts the mandible
Q1289:masseter
elevates and protracts the mandible
Q1290:sternocleidomastoid
head rotates; flexion of the head
Q1291:Implantation of blastocyst is present in which week?
1
Q1292:The bilaminar disk is formed within which week?
2
Q1293:Gastrulation; primitive streak; notochord; neural plate
begin to form
Within week 3
Q1294:Neural tube formed; organogenesis; extremely
susceptible to teratogens
Weeks 3-8
Q1295:Heart begins to beat; Upper and lower limb buds begin
to form
Week 4
Q1296:When do genitalia begin to have male/female
characteristics?
Week 10
Q1297:From day 0 to implantation at end of week 1; name
stages of embryogenesis
From day 0 to implantation at end of week 1; name stages of
embryogenesis
Q1298:For next 9; name effect on fetus of following
teratogens;ACE Inhibitors
Renal damage
Q1299:Cocaine
Abnormal fetal development; fetal addiction
Q1300:DES
vaginal clear cell adenocarcinoma
Q1301:Iodide
congenital goiter or hypothyroidism
Q1302:13-cis-retinoic acid
extremely high risk for birth defects
Q1303:thalidomide
thalidomide
Q1304:warfarin; x-rays
multiple anomalies
Q1305:fetal infections
congenital malformations
Q1306:how many umbilical arteries?
2
Q1307:what kind of blood do the umbilical arteries carry?
deoxygenated blood from fetus
Q1308:how many umbilical veins?
1
Q1309:what kind of blood is carried by umbilical vein?
supplies oxygenated blood from placenta to the fetus
Q1310:a single umbilical artery is associated with what
anomalies?
congenital and chromosomal
Q1311:What dervies from the following embyologic
layers;Surface Ectoderm
adenohypophysis; lens of eye; epithelial linings; epidermis
Q1312:neuroectoderm
neurohypophysis; CNS neurons; oligodendrocytes;
astrocytes; pineal gland
Q1313:neural crest
ANS; dorsal root ganglia; melanocytes; chromaffin cells of
adrenal medulla; enterochromaffin cells; pia; celiac ganglion;
Schwann cells; odontoblasts; parafollicular - C cells of
thyroid; laryngeal cartilage
Q1314:mesoderm
dura connective tissue; muscle; bone; cardiovascular
structures; lymphatics; blood urogenital structures; serous
linings of body cavities (peritoneal); spleen; adrenal cortex
Q1315:endoderm
gut tube epithelium and derivatives (lungs; liver; pancreas;
thymus; thyroud; parathyroid)
Q1316:what induces ectoderm to form neuroectoderm (neural
plate)?
notochord
Q1317:what is the postnatal derivative of the notochord?
nucleus pulposus of the intervertebral disk
Q1318:Name the precursor for the ectoderm and its purpose
epiblast; invaginates to form primitive streak
Q1319:Cells from the primitive streak give rise to
intraembryonic mesoderm and endoderm
Q1320:How many germ layers in second week and what are
they?
2; epiblast and hypoblast (remember rule of 2's for second
week)
Q1321:How many cavities in second week and what are they?
2; amniotic cavity and yolk sac (remember rule of 2's for
second week)
Q1322:How many components to placenta in second week
and what are they?
2; cytotrophoblast and syncytiotrophoblast (remember rule
of 2's for second week)
Q1323:How many germ layers in third week and what are
they?
3; (gastrula): ectoderm; mesoderm; endoderm (Rule of 3's for
third week)
Q1324:In the 3-8th weeks; fetal erythropoiesis occurs in
Yolk sac
Q1325:In the 6-30th weeks; fetal erythropoiesis occurs in
Liver
Q1326:In the 9-28th weeks; fetal erythropoiesis occurs in
Spleen
Q1327:Erythropoiesis occurs in the Bone marrow from the ---
- week onward
28th
Q1328:Mnemonic for remembering fetal erythropoiesis?
Young Liver Synthesizes Blood
Q1329:Embryonic structure gives rise to;Truncus arteriosus
Ascending aorta and pulmonary trunk
Q1330:Bulbus cordis
Smooth parts of left and right ventricle
Q1331:Primitive ventricle
trabeculated parts of left and right ventricle
Q1332:Primitive atria
trabeculated left and right atrium
Q1333:Left horn of sinus venosus (SV)
coronary sinus
Q1334:Right horn of sinus venosus (SV)
smooth part of right atrium
Q1335:Right common cardinal vein and right anterior cardinal
vein
SVC
Q1336:Saturation level of blood in umbilical vein?
80%
Q1337:Foramen ovale: its function in fetal circulation?
Most oxygenated blood reaching the heart via the IVC is
diverted through the foramen ovale and pumped out the aorta
to the head.
Q1338:Ductus Arteriosus: its function in fetal circulation?
Deoxygenated blood from the SVC is expelled into the
pulmonary artery and ductus arteriosus to the lower body of
the fetus.
Q1339:What happens at birth; when infant takes a breath?
Decreased resistance in pulmonary vasculature causes
increased left atrial pressure vs right atrial pressure; foramen
ovale closes; increase in O2 leads to decrease in
prostaglandins; causing closure of ductus arteriosus.
Q1340:What drug can be given to close a patent ductus
arteriosus?
Indomethacin
Q1341:What drug can be given to keep a patent ductus
arteriosus open?
Prostaglandins
Q1342:Fetal-postnatal derivatives;Umbilical vein becomes
the:
ligamentum teres hepatis
Q1343:umbilical arteries become the:
medial umbilical ligaments
Q1344:ductus Arteriosus becomes the:
ligamentum arteriosum
Q1345:ductus venosus becomes the:
ligamentum venosum
Q1346:foramen ovale becomes the:
fossa ovalis
Q1347:allantois - urachus becomes the:
median umbilical ligament
Q1348:notochord becomes the:
nucleus pulposus
Q1349:urachal cyst or sinus is a remnant of the:
allantois (urine drainage from the bladder)
Q1350:Aortic Arch Derivatives;1st arch:
part of maxillary artery (1st is MAXimal)
Q1351:2nd arch:
stapedial artery and hyoid artery (Second = Stapedial)
Q1352:3rd arch:
common Carotid artery and proximal part of internal carotid
artery (C is the 3rd letter of the alphabet)
Q1353:4th arch:
on left; aortic arch; on right; proximal part of right subclavian
artery 4th arch (4 limbs) = systemic
Q1354:6th arch:
proximal part of pulmonary arteries and (on left only) ductus
arteriosus. 6th arch = pulmonary and the pulmonary-to-
systemic shunt (ductus arteriosus)
Q1355:branchial clefts are dervied from:
ectoderm
Q1356:branchial arched are derived from:
mesoderm and neural crests
Q1357:branchial pouches are derived from:
endoderm
Q1358:mnemonic to remember branchial apparatus derivation:
CAP covers outside from inside (Clefts = ectoderm; Arches =
mesoderm; Pouches = endoderm)
Q1359:Branchial arch 1 derivatives;Meckel's cartilage:
Mandible; Malleus;incus;sphenoMandibular ligament
Q1360:Muscles:
Muscles of Mastication (temporalis; Masseter; lateral and
Medial pterygoids); Mylohyoid; anterior belly of digastric;
tesnsor tympani; tensor veli palatini
Q1361:Nerve:
CN V3
Q1362:Branchial Arch 2 derivatives;Reichert's cartilage:
Stapes; Styloid process; lesser horn of hyoid; Stylohyoid
ligament
Q1363:Muscles:
muscles of facial expression; Stapedius;Stylohyoid; posterior
belly of digastric.
Q1364:Nerve:
CN VII
Q1365:Branchial arch 3 derivatives;Cartilage:
greater horn of hyoid
Q1366:Muscle:
stylopharyngeus (Think of pharnx: stylopharyngeus is
innervated by glossopharyngeal nerve.
Q1367:Nerve:
CN IX
Q1368:Branchial arch 4 to 6 derivatives;Cartilages:
thyroid; cricoid; arytenoids; cornicuate; cuneiform
Q1369:Muscles (4th arch):
mostly pharyngeal constrictors; cricothyroid; levator veli
palatini.
Q1370:5th arch:
makes no major developmental contributions
Q1371:Muscles (6th arch):
all intrinsic muscles of larynx except cricothyroid
Q1372:Nerve (6th arch):
CNX (recurrent laryngeal branch)
Q1373:Branchial arch innervation;Arch 1 derviatives are
supplied by:
CN V2 and V3
Q1374:Arch 2 derivatives are supplied by:
CN VII
Q1375:Arch 3 derivatives are supplied by:
CN IX
Q1376:Arch 4 and 6 derivatives supplied by:
CNX
Q1377:Branchial cleft derivatives;1st cleft develops into
external auditory meatus
Q1378:2nd through 4th clefts form:
temporary cervical sinuses; which are obliterated by
proliferation of 2nd arch mesenchyme.
Q1379:Persistent cervial sinus can lead to a:
branchial cyst in the neck
Q1380:Ear development;Bone: Incus/Malleus come from:
1st arch
Q1381:Bone: Stapes comes from:
2nd arch
Q1382:Muscle: Tensor tympani (V3) comes from:
1st arch
Q1383:Muscle: Stapedius (VII) comes from:
2nd arch
Q1384:External auditory meatus comes from:
1st cleft
Q1385:Eardrum; eustacian tube comes from:
1st pharyngeal membrane
Q1386:Branchial pouch derviatives;1st pouch develops into:
middle ear cavity; eustacian tube; mastiod air cells (1st pouch
contributes to endoderm-lined structures of ear)
Q1387:2nd pouch develops into:
epithelial lining of palatine tonsil
Q1388:3rd pouch (dorsal wings) develops into:
inferior parathyroids (3rd pouch contributes to 3 structures:
thymus; left and right inferior parathyroids)
Q1389:3rd pouch (ventral wings) develops into:
thymus
Q1390:4th pouch develops into:
superior parathyroids
Q1391:Aberrant development of 3rd and 4th pouches -->
DiGeorge's syndrome --> leads to T-cell deficiency (thymic
hypoplasia) and hypocalcemia (parathyroid glands)
Q1392:Thymus;Site of:
T-cell maturation
Q1393:Encapsulated or unencapsulated?
encapsulated
Q1394:which branchial pouch?
from epithelium of 3rd branchial pouches
Q1395:Lymphocyte origin?
lymphocytes are of mesenchymal origin
Q1396:cortex is dense with:
immature T cells
Q1397:medulla is pale with:
mature T cells and epithelial reticular cells and contains
Hassall's corpuscles. (think of the Thymus as "finishing
school" for T cells. They arrive immature and "dense" in the
cortex; they are mature in the medulla.
Q1398:What occurs at the corticomedullary junction?
Positive and negative selection
Q1399:Thyroid diverticulum arises from:
floor of primitive pharnyx; descends into neck.
Q1400:Connected to tongue by
thyroglossal duct; which normally disappears but may persist
as a pyramidal lobe of thyroid.
Q1401:Formen cecum is:
a normal remnant of the thyroglossal duct
Q1402:The most common ectopic thyroid tissue site is the:
Tongue!
Q1403:Tongue development;1st branchial arch forms the
anterior 2/3 of the tongue (thus sensation via CN V3; taste via
CNVII)
Q1404:3rd and 4th branchial arches form:
posterior 1/3 of the tongue (thus sensation and taste mainly
via CN IX; extreme posterior via CN X).
Q1405:Cranial nerves for taste?
CN VII; IX; X (solitary nucleus)
Q1406:Motor innervation is via CN?
CN XII
Q1407:define cleft lip
failure of fusion of the maxillary and medial nasal processes
Q1408:define cleft palate
failure of fusion of the lateral palatine processes; the nasal
septum; and/or the median palatine process.
Q1409:diaphragm is derived from:
Septum transversarium; Pleuroperitoneal folds; Body wall;
Dorsal mesentery of esophagus (Several Parts Build
Diaphragm. Diaphragm descends during develpoement but
maintains innervation from above C3-C5. "C3; 4; 5 keeps the
diaphragm alive."
Q1410:What is a hiatial hernia
abdominal contents may herniate into the throax due to
incomplete development of the diaphragm.
Q1411:What is Intramembranous bone formation
spontaneous bone formation without preexisting cartilage.
Q1412:What is Endochondral bone formation
ossification of carilaginous molds. Long bones form by this
type of ossification at 1o and 2o centers
Q1413:What is Meckel's diverticulum
persistence of the vitelline duct or yolk stalk. May contain
ectopic acid-secreting gastric muscosa and/or pancreatic
tissue.
Q1414:What is the most common congenital anomaly of the
GI tract?
Meckel's diverticulum
Q1415:Complications of Meckel's Diverticulum?
can cause bleeding or obstruction near the terminal ileum.
Q1416:Comparison of Meckel's Diverticulum to
Omphalomesenteric cyst?
Cystic dilatation of the vitelline duct.
Q1417:It is Associated with
intussusception and volvulus
Q1418:Mnemonic to remember Meckel's diverticulum
The five 2's: 2 inches long; 2 feet from the ileocecal valve; 2%
of population; commonly presents in first 2 years of life; may
have 2 epithelia.
Q1419:Parcreas is derived from the
foregut
Q1420:ventral pancreatic bud becomes the
pancretic head; uncinate process (lower half of head); and
main pancreatic duct.
Q1421:dorsal pancreatic bud becomes:
everything else (body; tail; isthmus; and accessory pancreatic
duct).
Q1422:Spleen arises from
dorsal mesentery but is supplied by artery of foregut.
Q1423:Mesonephric (wolffian) duct develops into
Develops into Seminal vesicles; Epididymis; Ejaculatory duct;
and Ductus deferens (SEED)
Q1424:Paramesonephric (mullerian) duct;develops into
Develops into fallopian tube; uterus; and part of vagina.
Q1425:Mullerian inhibiting substance is secreted by:
the testes to suppress development of paramesonephric ducts
in males.
Q1426:Bicornuate uterus;results from and is Associated with
incomplete fusion of the paramesonephric ducts;urinary tract
abnormalities and infertility
Q1427:Genital tubercle (male) -->
glans penis via dihydrotestosterone
Q1428:genital tubercle (female) -->
glans clitoris via estrogen
Q1429:urogenital sinus (male) -->
corpus spongiosum; bulbourethral glands (of Cowper);
prostate gland: via dihydrotestosterone
Q1430:urogenital sinus (female) -->
vestiubular bulbs; greater vestibular glands (of Bartholin);
Urethral and paraurethral glands (of Skene): via estrogen
Q1431:urogenital folds (male) -->
ventral shaft of penis (penile urethra) via dihydrotestosterone
Q1432:urogenital folds (female) -->
labia minora via estrogen
Q1433:labioscrotal swelling (male) -->
scrotum via dihydrotestosterone
Q1434:labioscrotal swelling (female) -->
labia majora via estrogen
Q1435:Congenital penile abnormalities;Hypospadia
abnormal opening of the penile urethra on inferior (ventral)
side of penis due to failure of urethral folds to close.
Q1436:Epispadia
abnormal opening of the penile urethra on superior (dorsal)
side of penis due to faulty positioning of the genital tubercle.
Q1437:epispadia is associated with
exstrophy of the bladder
Q1438:Which is more common; hypospadia or epispadia?
Hypospadial; fix hypospadias to prevent UTI's
Q1439:Derivation of sperm parts;acrosome is derived from
the golgi apparatus and flagellum (tail) from one of the
centrioles.
Q1440:what does the neck of the sperm have?
Middle piece (neck) has Mitochondria.
Q1441:Sperm food supply is:
fructose
Q1442:Primary oocytes begin and complete meiosis I when?
begin: during fetal life; complete: just prior to ovulation.
Meiosis I is arrested in prOphase for years until Ovulation.
Q1443:What phase is Meiosis II arrested in?
Meiosis II is arrested in METaphase until fertilization. -- "An
egg MET a sperm."
Q1444:What is polyhydramnios?
high amount of amniotic fluid (>1.5-2L)
Q1445:What is polyhydramnios clinically associated with?
esophageal/duodenal atresis and anencephaly both of which
impair the ability of the fetus to swallow amniotic fluid
Q1446:What is the condition leading to low amniotic fluid
(<0.5L) called?
Oligohydramnios
Q1447:What 2 conditions is oligohydramnios associated
with?
Bilateral renel agenesis or posterior urethral valves (in males)
which results in the failure of the fetus to excrete urine into
the amniotic fluid.
Q1448:What syndrome can oligohydramnios result in?
Potter's Syndrome
Q1449:What does the term Potter's syndrom describe?
Bilateral renal agenesis leading to oligohydramnios which
results in fetal limb and facial deformaties and pulmonary
hypoplasis
Q1450:What causes Potter's syndrome?
Malformation of the ureteric bud
Q1451:What results when the poles of both kidneys fuse
during development?
Horseshoe kidney
Q1452:What arrests the ascend of the fused kidneys?
The horseshoe kidneys get trapped under the inferior
mesenteric artery.
Q1453:Which muscles form the rotator cuff?
Supraspinatus; Infraspinatus; Teres minor; and Subscapularis
(SItS)
Q1454:Which muscle helps the deltoid abduct the arm?
Supraspinatus
Q1455:What function does the Infraspinatus muscle have?
laterally rotates the arm at the shoulder joint
Q1456:What is the function of the Teres minor muscle?
to adduct and laterally rotate the arm (Possible mistake in
First Aid)
Q1457:Which muscle medically rotates and adducts the arm?
Subscapularis
Q1458:Name the thenar muscles.
Opponens pollicis; Abductor pollicis brevis; Flexor pollicic
brevis
Q1459:Name the hypothenar muscles.
Opponens digiti minimi; Abductor digiti minimi; Flexor digiti
minimi
Q1460:What functions do the thenar and hypothenar muscles
preform?
oppose; abduct and flex (OAF)
Q1461:What structures can be injured when a football
player's cleated shoe is planted firmly in the turf and the knee
is struck from the lateral side?
medial collateral ligament (MCL); medial meniscus; and
anterior cruciate ligament (ACL) (unhappy triad on the knee
joint)
Q1462:What does a anterior drawer sign indicate?
Tearing of the Anterior cruciate ligament (ACL)
Q1463:What does abnormal passive abduction indicate?
A torn Medial collateral ligament (MCL)
Q1464:The recurrent laryngeal nerve is a branch of which
cranial nerve?
CN X
Q1465:Which muscles does it supply?
intrinsic muscles of the larynx except the cricothyroid muscle
Q1466:What structure does the right recurrent laryngeal nerve
wrap around?
right subclavian artery
Q1467:What structure does the left recurrent laryngeal nerve
wrap around?
arch of the aorta and the ligamentum arteriosum
Q1468:In what kind of surgery can this nerve be damaged in?
thyroid surgery
Q1469:What happens when this nerve gets damaged?
hoarseness
Q1470:Name the layers of the scalp
skin; connective tissue; aponeurosis; loose connective tissue;
pericranium (SCALP)
Q1471:Name the meninges.
Dura; arachnoid; and pia (DAP)
Q1472:What space is found between the dura and arachnoid?
subdural space
Q1473:Between what meninges is the subarachnoid space
located?
between the arachnoid and the pia
Q1474:What is found in the subarachnoid space?
CSF
Q1475:In which scalp layer are the emissary veins found?
loose connective tissue
Q1476:List the structures found in the carotid sheath (in the
order from lateral; medial; posterior)
Internal jugular vein; common carotid artery; and vagus nerve
(VAN)
Q1477:What structure crosses the diaphragm at T8?
IVC (I 8 10 EGG's AT 12)
Q1478:At what T level does the esophagus and vagus nerve
cross the diaphragm?
T10 (I 8 10 EGG's AT 12)
Q1479:What structure crosses the diaphragm at T12?
aorta; thoracic duct; and azygous vein (I 8 10 EGG's AT 12)
Q1480:Name the innervation of the diaphragm
C3; 4; and 5 (phrenic nerve)
Q1481:Where can the pain from the diaphragm be reffered to?
shoulder
Q1482:What artery supplies the SA and AV nodes?
RCA right coronary artery
Q1483:The inferior portion of the left ventricle is supplied by
what artery 80% of the time?
RCA via the PD posterior descending artery
Q1484:In which artery does coronary artery occlusion most
sommonly occur?
LAD left anterior descending artery
Q1485:What artery supplies the anterior interventricular
septum?
LAD left anterior descending artery
Q1486:When do coronary arteries fill?
during diastole
Q1487:What is the most posterior part of the heart?
left atrium
Q1488:What can the enlargement of the left atrium cause?
dysphagia
Q1489:What does each bronchopulmonary segment contain?
3? (segmental) bronchus and 2 arteries (bronchial and
pulmonary) in the center
Q1490:What drains along the borders of the
bronchopulmonary segments?
veins and lymphatics
Q1491:What runs with the airways?
arteries
Q1492:How many lobes does the right lung has?
3 lobes
Q1493:Which lung lobe has 2 lobes?
left
Q1494:What is the homologue of the right middle lobe in the
left lobe?
lingula
Q1495:Which lung is the more common site for inhaled
foregin body?
right lung?
Q1496:Why is 1 lung a more common site for inhaled foregin
body?
Because of the lessvacute angle of the right main stem
bronchus
Q1497:What is the relation of the pulmonary artery to the
bronchus in each lung hilus?
RALS - right lung anterior and left lung superior
Q1498:What does the femoral sheath contain?
femoral artery; femoral vein; and femoral canal (containing
deep inguinal lymph node)
Q1499:Does the femoral nerve lie within the femoral sheath?
no; it lies outside the sheath
Q1500:What do you call the entrance of abdominal contents
through the femoral canal?
femoral hernia
Q1501:What does the femoral triangle contain?
femoral vein; aretery and nerve (VAN)
Q1502:Where does the femoral hernia protrude to?
below and lateral to the pubic tubercle
Q1503:What are abdominal hernias?
protrusions of peritoneum through an opening - usually sites
of weakness
Q1504:What is the name of the hernia in which abdominal
structures enter the thorax?
diaphragmatic hernia
Q1505:What is the most common diaphragmatic hernia?
hiatal hernia
Q1506:What happens in this type of hernia?
the stomach herniates upward through the esophageal hiatus
Q1507:What can result from a defective development of
pleuroperitoneal membrane in infants?
diaphragmatic hernias
Q1508:From where does a direct hernia protrude?
from the inguinal (Hesselbach's) triangle - bulges directly
through the abdominal wall medial to the inferior epigastric
artery
Q1509:Which inguinal ring does it go through?
external inguinal ring only
Q1510:Who usually gets a direct hernia?
older men
Q1511:What does the indirect hernia go through?
the internal (deep) inguinal ring and external (superficial)
inguinal ring and into the scrotum
Q1512:On which side of the inferior epigastric artery does the
indirect hernia enter the internal inguinal ring?
lateral to the inferior epigastric artery
Q1513:Who usually get an indirect hernia and why?
infants; because of the failure of processus vaginalis to close
Q1514:Which structures make up Hesselbach's triangle?
inferior epigastric artery; lateral border of rectus abdominis;
and inguinal ligament
Q1515:Where does the inguinal canal start and end?
begins at the deep inguinal ring and terminates at the
superficial ring
Q1516:What does the inguinal canal transmit?
the spermatic cord or the round ligament of the uterus and the
genital branch of the genitofemoral nerve
Q1517:What structures make up the anterior wall of the
inguinal canal?
aponeuroses of the external oblique and internal oblique
muscles
Q1518:What structures make up the posterior wall of the
inguinal canal?
aponeurosis of the transverse abdominal muscle and
transversalis fascia
Q1519:What structures make up the superior wall of the
inguinal canal?
arching fibers of the internal oblique and transverse muscles
Q1520:What structures make up the inferior wall (floor) of
the inguinal canal?
inguinal and lacunar ligaments
Q1521:From where does the stomach receive its main blood
supply?
from the branches of the celiac trunk
Q1522:From where does the celiac trunk arise?
from the front of the abdominal artery immediately below the
aortic hiatus of the diaphragm
Q1523:What arteries does the celiac trunk divide into?
left gastric; splenic; and common hepatic arteries
Q1524:What does the left gastric artery run along?
lesser curvature of the stomach
Q1525:What artery does the left gastric artery anastomose
with?
the right gastric artery
Q1526:What does the splenic artery run along?
runs along the superior boarder of the pancrease
Q1527:What arteries does the splenic artery give rise to?
(dorsal pancreatic artery); short gastric arteries and left
gastroepiploic artery
Q1528:What does the left gastroepiploic artery run along?
the greater curvature of the stomach
Q1529:What does the common hepatic artery divide into?
hepatic artery proper; right gastric artery; and gastroduodenal
artery
Q1530:esophageal varices results at the anastamosis of which
veins?
left gastric vein (portal) and azygous (systemic)
Q1531:manifestation of portal hypertension at the
anastamosis of the superior rectal with the middle/inferior
rectal veins
hemorrhoids
Q1532:caput medusae is at anastamosis of which veins
paraumbilical (portal) and inferior epigastric (systemic)
Q1533:the two other sites of portal system anastamosis
retroperitoneal vein (portal) with renal vein (systemic) and
retroperitoneal vein with paravertebral vein (systemic)
Q1534:3 manifestations seen in alcoholic cirrhosis
esoph. Varices; hemorrhoids; caput medusae "Gut; butt and
caput"
Q1535:What drains the lymph from right arm and right half of
head
right lymphatic duct
Q1536:What does the Thoracic duct drain
everything that right lymphatic duct does not
Q1537:location of pectinate line
where hindgut meets ectoderm
Q1538:innervation; hemorrhoid type; aterial supply and
venous drainage above pectinate line
visceral innervation(not painful); internal hemorrhoids (think
of adenocarcinoma association); superior rectal artery(branch
of IMA); and superior rectal vein to inferior mesenteric vein
to portal system
Q1539:innervation; hemorrhoid type; aterial supply and
venous drainage below pectinate line
somatic innervation(painful!); external hemorrhoids (think of
squamous cell carcinoma association); inferior rectal artery;
inferior rectal vein to internal pudendal vein to internal iliac
vein to IVC
Q1540:parts of duodenum that are retroperitoneal
2nd; 3rd and 4th parts
Q1541:parts of colon that are retroperitoneal
descending and ascending colon
Q1542:2 big organs that are retroperitoneal
pancreas (except tail) and kidneys
Q1543:2 major vessels that are retroperitoneal
aorta and IVC
Q1544:two big nerve plexuses in digestive tract and what they
do
Submucosal (Meissners) controls Secretions; Myeneteric
(Auerbach's) controls Motility
Q1545:Layers of digestive tract from inside to out
villi; lamina propria; muscularis mucosae; submucosa; serosa
Q1546:venous drainage of left ovary/testis
left gonadal vein to left renal vein to IVC
Q1547:venous drainage of right ovary/testis
right gonadal vein to IVC
Q1548:location of myenteric plexus
b/w inner and outer layers of GI smooth muscle
Q1549:location of submucosal plexus
b/w mucosa and inner layer of GI smooth muscle
Q1550:region and structures supplied by celiac artery
foregut: stomach to duodenum; liver; gallbladder; pancreas
Q1551:hindgut is supplied by which artery
inferior mesenteric artery
Q1552:superior mesenteric artery supplies ---
midgut: duodenum to proximal 2/3 of transverse colon
Q1553:Kidney anatomy and glomerular structure;grossly; the
collecting system is made up of --
papillae; medullary pyramids; renal pelvis and ureter
Q1554:which structures are in the renal medulla?
proximal and distal straight tubules; loop of Henle; vasa recta
Q1555:location of macula densa
part of DCT next to afferent arteriole
Q1556:two components of JGA
JG cells (afferent arteriole) and macula densa (DCT; senses
Na)
Q1557:three factors leading to renin secretion
decreased renal blood pressure; decreased sodium in distal
tubule; increased sympathetic tone
Q1558:what secretes erythropoeitin?
JG cells
Q1559:two structures under which ureters pass
uterine artery and ductus deferens "water under bridge"
Q1560:Ligaments of the uterus ;which ligament contains
ovarian vessels?
suspensatory ligament of uterus
Q1561:what does round ligament of uterus contain?
nothing!
Q1562:which ligament contains uterine vessels?
transverse cervical (cardinal) ligament
Q1563:what does broad ligament contain?
round ligaments of uterus and ovaries and uterine tubules and
vessels
Q1564:erection is mediated by ---- while emission is mediated
by ----.
Parasympathetics; Sympathetics "Point and Shoot"
Q1565:which nerves mediate ejaculation?
visceral and somatic
Q1566:Clinically important landmarks;McBurney's point
site of appendix; 2/3 of way from umbilicus to ASIS
Q1567:significance of ischial spine
site of pudendal nerve block
Q1568:site of lumbar puncture
iliac crest
Q1569:Peripheral nerve layers;what is covered by
endoneurium?
single nerve fiber
Q1570:what must be rejoined in microsurgery for limb
reattachment?
Perineurium: the Permeability barrier
Q1571:what surrounds the entire nerve?
epineurium
Q1572:Collagen types;most abundant protein in human body
collagen
Q1573:collagen primarily found in basement membrane and
basal lamina
type IV. "under the floor"
Q1574:collagen type primarily found in cartilage
type II. "carTWOlage"
Q1575:collagen type primarily found in bone
type I. "bONE"
Q1576:Reticulin found in skin; blood vessels; uterus; fetal
tissue; and granulation tissue
type III.
Q1577:Collagen type X is found primarily in -----.
epiphyseal plate
Q1578:What are the layers of the epidermis from surface to
base?
stratum corneum; stratum lucidum (in thick skin); stratum
granulosum; stratum spinosum; stratum basalis. "Californians
Like Girls in String Bikinis"
Q1579:This type of junction is also called the zona occludens.
(occludes diffusion across intracellular space)
tight junction
Q1580:Forming a perimeter just below the tight junction; the
zona adherens (intermediate junction) is composed of ------
and ------ filaments.
e-cadherin & actin filaments
Q1581:Desmoplakin and ----- filaments are a part of the
desmosome; also called the ----- ------.
keratin. Macula adherens
Q1582:Structures that allow adjacent cells to communicate for
electric and metabolic purposes.
gap junction
Q1583:These structures connect cells to underlying
extracellular matrix. Integrins are found in these structures.
hemidesmosome
Q1584:The glomerular basement membrane is formed from
fused endothelial and podocyte basement membranes and
coated with ---- (pos / neg) charged ------ ------.
negatively charged heparan sulfate
Q1585:Responsible for filtering plasma according to ----- and
----.
charge & size.
Q1586:In Nephrotic syndrome; Negative charge is lost. That's
why plasma -----; which are negatively charged; are lost in the
urine as a consequence.
protein
Q1587:Principle antigen presenting cell of epidermis
Langerhans cell
Q1588:what is the arrangement of microtubules?
9+2
Q1589:what is dynein?
An ATPase that links peripheral 9 doublets and causes
bending of cilium by differential sliding of doublets
Q1590:what happens in Kartagener's syndrome?
due to a dynein arm defect; resulting in immotile cilia
Q1591:What are nissl bodies?
RER in neurons; not found in axon or axon hillock
Q1592:What do nissl bodies synthesize?
enzymes (i.e. ChAT) and peptide neurotransmitters
Q1593:What is I-Cell Disease
Failure of golgi to add mannose-6-phos to lysosome proteins;
causing their secretion outside the cell (defect of golgi)
Q1594:name organelle of protein synthesis
Rough ER
Q1595:name organelle of protein transport
Golgi
Q1596:name organelle of N-oligodsaccharide addition
Rough ER
Q1597:name organelle of N-oligodsaccharide modification
Golgi
Q1598:name organelle of proteoglycan assembly
Golgi
Q1599:name organelle of steroid synthesis and detoxification
of drugs and poisons
Smooth ER
Q1600:Mucus-secreting goblet cells of small intestine and
antibody secreting plasma cells are rich in ______ (organelle)
Rough ER
Q1601:Liver hepatocytes and steroid producing cells of the
adrenal cortex are rich in ______ (organelle)
Smooth ER (look at function two questions above)
Q1602:T or F: Liver sinusoids have basement membranes?
Falso
Q1603:T or F: Spleen sinusoids have basement membranes?
True -- "barrel hoop" basement membranes line sinusoids
Q1604:Spleen sinusoids traverse the _______ pulp
Red
Q1605:Match: T-cells are in the red or white pulp
Red
Q1606:Match: B-cells are in the red or white pulp
White (within the follicles)
Q1607:T or F: alpha cells make glucagon
True
Q1608:T or F: B cells make somatostatin
False -- beta cells make insulin
Q1609:T or F: delta cells make somatostatin
TRUE
Q1610:Islet cells arise from what primordial structure
Pancreatic Buds
Q1611:Zona Glomerulosa makes ________
Aldosterone
Q1612:What zona makes the sex hormones
Zona reticularis
Q1613:What zona makes cortisol
Zona Fasciculata (remember GFR and "The Deeper you go;
the sweeter it gets" -- i.e. salt; sugar; sex
Q1614:What is the primary regulator of the zona fasciculata
ACTH (direct) and CRH (indirect)
Q1615:What is the primary regulator of the zona glomerulosa
Renin-Angiotensin
Q1616:what is the primary regulator of the zona reticularis
ACTH (direct) and CRH (indirect)
Q1617:What is the most common tumor of the adrenal
medulla in adults
Pheochromocytoma
Q1618:What is the most common tumor of the adrenal
medulla in children
Neuroblastoma
Q1619:What is the main secretory product of the medulla
Catecholamines
Q1620:What is the main secretory product of Brunner's
Glands and where do they reside
Secrete alkaline mucus and live in the submucosa of the
duodenum
Q1621:Is a lymph node a primary or secondary lymphoid
organ
Secondary
Q1622:What part of the lymph node is the site of B-cell
localization and proliferation
Follicle
Q1623:What is the function of the medulla
Communicate with efferent lymphatics and contain
lymphocytes; plasma cells; macs; and reticular cells
Q1624:The paracortex is the home of ____ cells
T-cells
Q1625:What is the name of the syndrome associated with
disfunctional paracortex
DiGeorge's Syndrome
Q1626:What tissue is responsible for IgA secretion in the gut?
Peyer's patch (Think IgA -- Intra-Gut Antibody) --
Stimulated B-cells from Peyer's secrete IgA into the lumen
Q1627:At what level should one perform a lumbar puncture?
Between L4 and L5 (to keep the cord alive; keep the needle
between L3 and L5)
Q1628:What structures are pierced?
Skin; Ligaments; Epidural space; Dura; Subdural Space;
Arachnoid; Subarachnoid (CSF) (NOTE: pia is NOT pierced)
Q1629:Appearance of nucleous on micrograph
Nuclear envelope continous with RER; euchromatin is white;
heterochromatin is dark; central area of nucleolus is light and
has DNA not being transcribed; peripheral granular dark area
of nucleoulus contains ribosomal precursors in different stages
of assembly.
Q1630:30nm solenoid chromatin fibers
Composes heterochromatin and its inactive. Nucleosomes
plus H1 molecules
Q1631:10nm solenoid chromatin fibers
Compose euchromatin and its active. Histone octamers
(nucleosomes)
Q1632:Nucleosome
Composed of lysine/arginine-rich histone octamers with
negatively-charged DNA wrapped around it.
Q1633:Nuclear lamina
In the inner side of the nuclear envelope. Phosphorylation of
the lamina disassembles nucleus during prophase.
Q1634:Function of the nucleus
Replication and transcription of DNA. Ribosomal 40S
subunit synthesis and assembly.
Q1635:Function of the nucleolus
rRNA synthesis and ribosomal assembly of 60S subunits
Q1636:Functions of RER
Synthesis of proteins destined to be secreted; lysosomes and
cell membrane
Q1637:Functions of SER
Detoxification reactions (steroid synthesis; hydroxylation via
cytochrome P450; conjugation); Glycogenolysis and
gluconeogenesis; Lipid metabolism; sequestration of Ca+.
Q1638:Functions of Golgi apparatus
Posttranslational modifications (acetylation; gamma-
carboxylation) Formation of glycoproteins and oligossachride
chains (phosphorylation of mannose oligosacchride residues
and I-cell disease; insulinase cleaves proinsulin into insulin
and C-peptide)
Q1639:I-cell disease
No phosphorylation of mannose residues in oligossacchride
chains of glycoproteins fails to direct hydrolases to
lysosomes. They are secreted instead. Skeletal abnormalities;
coarse facial features; mental retardation.
Q1640:Functions of lysosomes
Contain acid hydrolases for degradation of diverse substances.
Hydrolases are assembled in the RER and modified in the GA
by phosphorylation of mannose residues.
Q1641:Functions of peroxisomes
Synthesis and degradation of H2O2; Beta oxidation of very
long chain fatty acids; Bile acid synthesis.
Q1642:Structure of mitochondria
Outer membrane (permeable); Inner membrane with cristae
(impermeable; contain oxidative phosphorylation and electron
transport enzymes); Matrix (contain dehydrogenases -
pyruvate; fatty acids).
Q1643:Function of mitochondria
Oxidative phosphorylation; electron transport and generation
of ATP; beta oxidation; part of the urea cycle and
gluconeogenesis
Q1644:Functions of microtubules
Chromosomal movement during meiosis and mitosis;
Intracellular organelle transport (kinesin; dynein); cilliary and
flagellar movement.
Q1645:Chediak-Higashi
Defect of microtubule polymerization. Prevents phagocytosis
(delayed phagosome-lysosome fusion); partial albinism
(increased fusion of melanosomes in melanocytes); granule
accumulation in NK cells and platelets.
Q1646:Functions of microfilaments
Composed of actin. Local movements
(polymerization/depolymerization) and sliding movements via
interaction with myosin
Q1647:Composition of the basement membrane
Type IV collagen; glycoproteins (laminin); proteoglycans
(heparan sulfate)
Q1648:Tight junctions (zonula occludens)
Fusion of opposed membranes in the apical borders. Seals off
the intercellular space. Forms barriers in epithelium.
Q1649:Zonula adherens
Basal to the tight junctions. Attaches adjacent epithelial cells.
Q1650:Desmosomes (macula adherens)
Juxtaposed spot welds that originate within the cytoplasm of
two adjacent cells and hold them together.
Q1651:Gap junctions
Tubules that connect and allow pasage of ions between cells.
Q1652:What are the apical surface specializations?
Microvilli; cilia; stereocilia
Q1653:Functions of cilia
Propel fluid and substances in one direction; clear mucus from
respiratory tract
Q1654:Kartagener syndrome
Absent or defective dynein arms of cilia. Chronic sinusitis;
bronchiectasis; infertility; situs inversus.
Q1655:Axon hillock
Interfase between neuron body and axon where depolarization
begins.
Q1656:Unmyelinated axons
In peripheral nerves; surrounded by Schwann cells; small
diameter; slow conduction
Q1657:Myelinated axons
Large diameter; fast conduction
Q1658:Schwann cells
Myelinating cells of peripheral nervous system. Myelination
begins in the fourth month.
Q1659:Oligodendrocytes
Myelinating cells of CNS. Myelination begins in the 4th
month until the 2nd decade of life. Degeneration of
oligodendrocytes produces multiple sclerosis and
demyelinating diseases.
Q1660:Node of Ranvier
Unmyelinated segment in between myelinated segments of the
axon. The action potential skips from node to node.
Q1661:Astrocytes
Maintain the blood-brain barrier
Q1662:Microglia
Derive from mesoderm and are phagocytes of the
mononuclear system
Q1663:Ependymal cells
Located in the ventricles; have cilia to move CSF.
Q1664:Structure of the sarcomere
I band with Z line in between; A band with H band in
between. H band contains thick myosin filaments. I band
contains thin actin filaments. A band contains thick and thin
filaments. H and I bands shorten during contraction.
Q1665:Actin
Double helix component of thin filaments; has myosin
crossbridges binding sites
Q1666:Tropomyosin
Double helix component of thin filaments that run on actin
filaments covering the myosin binding sites
Q1667:Troponins
TnT binds tropomyosin; TnC binds calcium; TnI inhibits
actin-myosin interaction. When calcium binds TnC; the
troponin-tropomyosin complex moves and uncovers the actin
binding sites for myosin crossbridges.
Q1668:Myosin
Component of the thick filaments. Has a head and two tails.
The head has ATPase activity and actin-binding sites to form
crossbridges.
Q1669:Lymph node cortex
B cells; follicles and germinal centers
Q1670:Lymph node paracortex
Populated by T cells
Q1671:Spleen white pulp
Surrounds arteries. Periarteriolar sheaths contain T cells and
peripheral white pulp contains B cells and germinal centers.
Q1672:Spleen red pulp
Is beyond the periarteriolar sheaths and contains macrophages
that extract damaged erythrocytes from blood.
Q1673:Layers of epidermis
Basale (columnar cells); spinosum (cuboidal); granulosum
(flat); lucidum (flat anucleated); corneum (flat kertinized)
Q1674:Keratinocytes
Produce keratin and have desmosomes. In pemphigus there
are autoantibodies against keratinocyte desmosomes.
Q1675:Melanocytes
Neural crest ectoderm derivatives. In dermis and epidermis
produce melanin melanosomes.
Q1676:Langerhans cells
Dendritic antigen presenting cells.
Q1677:Merkel cells
Sensory cells with cathecolamine granules.
Q1678:Pemphigus
Autoantibodies against keratinocyte desmosomes. Superficial
blisters.
Q1679:Psoriasis
Hyperplasia of basale and spinosum layers with increased
turnonver and thickness. Produces descamation.
Q1680:Albinism
Melanocytes won't produce melanin due to lack of tyrosinase.
Increased risk of SCC and melanomas.
Q1681:Vitiligo
Autoantibodies against melanocytes produce depigmentation
Q1682:Bullous pemphigoid
Autoantibodies against hemidesmosomes in dermis/epidermis
junction. Blisters and separation of dermis from epidermis.
Q1683:Dermis
Derived from mesoderm. Supplies blood to epidermis and
contains appendages.
Q1684:Eccrine sweat glands
Most body parts; watery discharge; cholinergic innervation.
Q1685:Apocrine sweat glands
Axila; areola and anal regions. Adrenergic innervation.
Q1686:Sebaceous glands
Associated with hair follicles. Produce lubrication for hair and
skin.
Q1687:Cells of the respiratory nasal cavities
Pseudostratified; ciliated; columnar with goblet cells; mucous
and serous glands. The conchae increase surface area; warms
up the air and is richly vascularized and innervated.
Q1688:Cells of the olfactory nasal cavities
In the posterosuperior nasal cavity; has pseudostratified
bipolar neuron which can divide. Bowman glands produces
serous fluids that dissolve odorous substances
Q1689:Paranasal sinuses
Frontal; maxillary (2); sphenoid and ethmoid
Q1690:Type I pneumocytes
Line the alveoli lumen and constitutes the main cell of the
blood-air interface
Q1691:Type II pneumocytes
Cuboidal cells that sit on the basal lamina and produce
surfactant to reduce alveolar surface tension.
Q1692:Alveolar macrophages
On the surface of alveoli. Phagocytosis of foreign particles.
Q1693:Esophagus histology
Nonkeratinized stratified squamous epithelium. Skeletal
muscle in upper 1/3; smooth muscle in lower 1/3.
Q1694:Barret's esophagus
Squamous-columnar metaplasia at gastro-esophageal junction
due to acid-reflux disease
Q1695:Cells of body and fundus of stomach
Mucous cells (secrete mucous; tight junctions); Chief cells
(pepsinogen; lipase); Parietal cells (IF; HCl)
Q1696:Cells of the pylorus
Mucous cells (mucous and tight junctions); Parietal cells.
Q1697:Characteristics of duodenum
Brunner glands secrete alkaline secretions; Goblet cells secrete
acid glycoproteins that protect mucosa; Paneth cells; EE cells
secrete CCK and secretin.
Q1698:Characteristics of jejunum
Brunner glands secrete alkaline secretions; Goblet cells secrete
acid glycoproteins that protect mucosa; Paneth cells; EE cells
secrete CCK and secretin.
Q1699:Characteristics of ileum
Peyer patches are lymph aggregations; M cells endocytose
antigen from lumen to lymphoid cells.
Q1700:Layers of the digestive tract
From the lumen out: mucosa; muscularis mucosa; submucosa;
muscularis externa; serosa
Q1701:Parotid gland
Serous acinars secrete amylase. Innervated by IX
Q1702:Visceral layer of Bowman's capsule
Podocytes rest on basal lamina which is shared with the
fenestrated capillary endothelium.
Q1703:Parietal layer of Bowman's capsule
Simple squamous epithelium continous with proximal tubule
epithelium.

Vous aimerez peut-être aussi